10-preptest 10 explsdl.keywin.org/1/e/1e1103afee7eee195fe5fbe511587657.pdfno part of this book may...

76
KAPLAN LSAT PREP LSAT RELEASED TEST X EXPLAINED A Guide to the February, 1994 LSAT KAPLAN The answer to the test question.

Upload: others

Post on 07-Sep-2020

0 views

Category:

Documents


0 download

TRANSCRIPT

Page 1: 10-PrepTest 10 Explsdl.keywin.org/1/e/1e1103afee7eee195fe5fbe511587657.pdfNo part of this book may be reproduced in any form, by photostat, microfilm, xerography or any other means,

KAPLAN LSAT PREP

LSAT

RELEASED TEST XEXPLAINED

A Guide to the February, 1994 LSAT

KAPLANThe answer to the test question.

Page 2: 10-PrepTest 10 Explsdl.keywin.org/1/e/1e1103afee7eee195fe5fbe511587657.pdfNo part of this book may be reproduced in any form, by photostat, microfilm, xerography or any other means,

1994 Stanley H. Kaplan Educational Center Ltd

All rights reserved. No part of this book may be reproduced in any form, byphotostat, microfilm, xerography or any other means, or incorporated into anyinformation retrieval system, electronic or mechanical, without the written permissionof Stanley H. Kaplan Educational Center Ltd.

Page 3: 10-PrepTest 10 Explsdl.keywin.org/1/e/1e1103afee7eee195fe5fbe511587657.pdfNo part of this book may be reproduced in any form, by photostat, microfilm, xerography or any other means,

© K A P L A N 1

SECTION I:

LOGICAL REASONING

Page 4: 10-PrepTest 10 Explsdl.keywin.org/1/e/1e1103afee7eee195fe5fbe511587657.pdfNo part of this book may be reproduced in any form, by photostat, microfilm, xerography or any other means,

KAPLAN LSAT PREP ________________________________________________________ LSAT Test X Explained: Section I

2 © K A P L A N

1. (B)In arguing that educational television is not educational, the author contrasts televisionwith the activity found in a classroom: Since television isn’t like school, it’s thereforenot educational. The argument therefore assumes that a school-like environment is aprerequisite for educational value.

(A) The author doesn’t suggest that fun in the classroom is inappropriate. In fact, byreferring to fun as a “means to an end,” the author hints that “fun” may, in some way,belong in the classroom.

(C) Inconsistent; the foundation of the argument is that television lacks a commonground with school.

(D) Beyond the scope of the argument. The only television programming discussed inthe argument is “educational” TV. At no point is any mention made of other sorts ofprogramming.

(E) The author argues that television is not an educational medium. That TV might be a“powerful learning tool” hardly seems consistent with this theme.

• Simplify logical arguments into their most basic form. Think of arguments inabstractions. For example: A is a good thing. A has certain properties. Since Blacks these properties, B must not be a good thing.

• Assumptions forge the “missing link” between premises and conclusions. Inassumption questions, be on the lookout for such logical leaps. The correctchoice will be the one that fills in the gap.

2. (E)The paradox we’re asked to resolve is how cigarettes containing fewer toxins still carrythe same toxic effects as regular cigarettes. Well, if “low-yield” cigarette smokers wereto change their smoking habits so as to maintain their normal nicotine levels, this wouldin fact negate the benefit of these cigarettes.

(A) suggests little more than the fact that many people may have switched fromtraditional cigarettes to the “low-yield” variety. However, it doesn’t address whethersmokers increase, decrease, or maintain their overall intake of toxins.

(B) is beyond the scope: We’re only concerned with the relationship between smokingcigarettes and heart disease, so the fact that something else (second-hand smoke) isn’t arisk factor has no bearing.

(C) Since nicotine and carbon monoxide are dealt with together in the description oflow-yield cigarettes, knowing that one is more dangerous than the other gets us nocloser to resolving the paradox.

(D) The issue of addictive substances in general is outside the scope.

Page 5: 10-PrepTest 10 Explsdl.keywin.org/1/e/1e1103afee7eee195fe5fbe511587657.pdfNo part of this book may be reproduced in any form, by photostat, microfilm, xerography or any other means,

KAPLAN LSAT PREP ________________________________________________________ LSAT Test X Explained: Section I

© K A P L A N 3

• Learn to recognize paradox questions. The question stem may use the word“paradox” itself or take the form of the stem of Question 2.

• Resolving paradoxes is often a matter of recognizing that two things that arebeing compared aren’t really the same thing. Read critically to note these subtledistinctions.

• In paradox questions, avoid choices that merely amplify points already raised inthe argument.

3. (E)Sally’s obvious motive for considering transfer to another school is to escape thealcohol problems that make it impossible for her to study. If we consider Sally’sargument without the benefit of the assumption in choice (E), the argument falls topieces: If (E) is false, then she’ll have no place to go to escape the problem. Since she isplanning to transfer, she must believe that there are alcohol-free universities somewhere(good luck, Sally).

(A) While she implies that there exist universities without frats, we can’t tell whethershe believes that most have them; either way, it’s irrelevant to the argument.

(B) Sally will transfer unless something is done about the problem. While her tone mayseem skeptical, Sally has recognized that the problem may be resolved.

(C) is irrelevant to Sally, whose only focus is to find an alcohol-free school if her currentschool doesn’t clean up its act.

(D) The classic beyond the scope choice: While Sally has linked the alcohol problemwith fraternities, she couldn’t care less whether the drinking is legal or illegal.

• Read the question stem first; in this case, doing so would have told you to focusonly on Sally for the sake of this question.

• If you are given a particular person’s perspective, make sure the choice you pickstays within that person’s scope.

4. (D)Sally’s transfer plans center around the theory that “no fraternities” equals “no alcoholproblem.” Yolanda attacks this argument by pointing out that alcohol abuse is apervasive cultural problem, and that therefore even schools with no fraternities stillhave alcohol problems.

(A) Although Yolanda advocates awareness of alcohol abuse, she never discusses theimpact such awareness will have on the problem.

Page 6: 10-PrepTest 10 Explsdl.keywin.org/1/e/1e1103afee7eee195fe5fbe511587657.pdfNo part of this book may be reproduced in any form, by photostat, microfilm, xerography or any other means,

KAPLAN LSAT PREP ________________________________________________________ LSAT Test X Explained: Section I

4 © K A P L A N

(B) Yolanda doesn’t generalize as (B) describes; in fact, she does the opposite. Shebegins with the general proposition that alcohol abuse is a cultural problem and thenapplies that generalization to the specific, i.e. universities.

(C) This choice confuses Yolanda’s point. She doesn’t argue that alcohol abuse isn’t afraternity problem; she merely points out that “it’s not simply a fraternity problem,”implying that alcohol abuse is a fraternity problem but one that’s not unique tofraternities.

(E) Yolanda makes no such suggestion. Her argument is that the problem exists at alluniversities. The premise in (E) is inconsistent with that.

• Note the difference between reasoning from the specific case to the general, andreasoning from the general to the specific. Stay away from choices that move inone direction while the stimulus argument moves in the opposite.

• In dialogues, be certain to isolate the argument each person is making. Read thesecond argument with respect to its relation to the first.

• Try not to skip stimuli that come with two questions attached. Often the secondquestion can be answered quickly after you’ve invested time in the first (or viceversa).

5. (B)The key phrase is “if the Association wants a mayor who will attract more business. . .,” which implies that it’s not a foregone conclusion that the Association wants that atall. The argument boils down to “if they want a specific thing, then Cooper is the onlyviable candidate, and since they’re supporting Cooper, then they must in fact want thisspecific thing.” It’s altogether possible, based on the wording of the argument, that theAssociation does not want a mayor who attracts more business, that they’re supportingCooper for other reasons — in which case this circular kind of reasoning falls flat.

(A) and (D) confuse the point of the argument, which seeks to explain why theAssociation supports a candidate, not the reasoning behind its goals or what’s in thebest interest of its members down the road.

(C) goes beyond the scope of the argument. We’re only concerned with theHomeowners Association, not with other groups.

(E) If the Association does in fact support Cooper for the reasons given, then this choiceaddresses a possible flaw in their decision. However, (E) doesn’t address the overridingflaw in the argument’s structure, and that’s the possibility stated in correct choice (B).

• In “flaw” questions, as you evaluate the argument, ask yourself, “assuming thatall the premises are true, is this the only conclusion that one could reach?” If theanswer is “no,” look for a choice that deals with the ambiguity.

• Before attempting to find a logical flaw, restate the argument in simple terms.

Page 7: 10-PrepTest 10 Explsdl.keywin.org/1/e/1e1103afee7eee195fe5fbe511587657.pdfNo part of this book may be reproduced in any form, by photostat, microfilm, xerography or any other means,

KAPLAN LSAT PREP ________________________________________________________ LSAT Test X Explained: Section I

© K A P L A N 5

• Be wary of choices that deal with people (or other groups) not mentioned in theargument.

6. (C)The advertisement reaches the conclusion that its product’s safety is “assured.”However, the only evidence offered on this point is that the product has been tested by areputable laboratory. If the test results were positive, then the conclusion would seemfairly justified. However, if the test results were negative, then the ad would be grosslydeceptive. The fact that the product has been tested is not enough; we need to know theresults of the tests to determine the ad’s truthfulness.

(A) Since the ad only deals with the safety of the hedge trimmer, knowing whetherother products have also been tested won’t bring us any closer to judging the ad’svalidity.

(B) How the consumers perceive the safety issue is one step away from the argument.Whether or not people regard safety as important, the hedge trimmer may still be safeor dangerous.

(D) Who cares? The ad assures the safety of a hedge trimmer; alternative ways oftrimming hedges are irrelevant.

(E) Knowing the answer to this question won’t help us evaluate the ad, because theadvertisement doesn’t claim that Bolter’s is the best for trimming hedges, only that itssafety is guaranteed.

• In “relevant information” questions, be on the lookout for a “missing link”: Ifthe question contains such a logical leap, the choice that bridges the gap may verywell be the answer.

• When asked for a question that would resolve an ambiguous issue, rememberthat you are not being asked to resolve the issue itself.

• If you focus exclusively on the issue at hand (here, the safety of Bolter’s hedgetrimmer is what’s assured), you’ll be less likely to get sucked in by outside-the-scope answer choices.

7. (A)If tropical forests do regenerate after they have been subjected to slash-and-burn, itwould be possible that while farmers move from one section of land to another,previously cleared land would ultimately grow back with forest vegetation. Thisdirectly conflicts with the conclusion that the forests would be permanently eradicated,so it must be assumed that the forests cannot regenerate well enough to offset slash-and-burn.

(B) At no point does the argument hint at the possibility that another agriculturalmethod would have a more positive — or negative — impact on tropical regions.

Page 8: 10-PrepTest 10 Explsdl.keywin.org/1/e/1e1103afee7eee195fe5fbe511587657.pdfNo part of this book may be reproduced in any form, by photostat, microfilm, xerography or any other means,

KAPLAN LSAT PREP ________________________________________________________ LSAT Test X Explained: Section I

6 © K A P L A N

(C) The growth of “foreign” plants has no bearing on the argument, so it cannot be anecessary assumption.

(D) and (E) The author doesn’t judge the utility of slash-and-burn — she’s concernedonly with the effect that method has on the environment, not its usefulness with respectto farming.

• An assumption bridges the gap between evidence and conclusion. It’s a piece ofinformation that the argument depends on for the conclusion to remain valid.

• Choices that are inconsistent with or neutral to the argument are wrong.

• The Denial Test can be used to check answers to assumption questions. Deny ornegate your choice and see if the argument falls apart. If it does fall apart, theanswer is correct. If the conclusion is unaffected, the choice is wrong.

8. (E)The flaw here is pretty subtle, and therefore difficult to pick out, but it centers aroundthe fact that the sample consists of 2,500 people who survived their first heart attack. Whyis this important? The conclusion gives figures relating to first heart attacks in general,but is based on evidence gathered from heart attack victims who survived — and ittotally ignores those who didn’t. The sample could therefore be biased towardsyounger people; after all, people in their 50’s having a first heart attack are more likelyto survive than people in their 80’s having their first heart attack. There could be manyof these cases (smokers and non-smokers alike) who aren’t represented in the sample ofthis study, which, if included, would significantly alter the results. Therefore, we can’tlogically arrive at the conclusion in the last sentence without information on this group.

(A) The basis of the argument is the timing of heart attacks; their severity is irrelevant.

(B), (C) The scope of this argument is limited to the timing of the first heart attack, sowhat occurs after the first attack is outside the scope of the argument.

(D) Knowing the earliest age at which someone had a heart attack doesn’t affect thisargument.

• When a conclusion is based on a statistical sample, be certain that the sample isrepresentative of the group for which the conclusion applies.

• When statistics appear in an argument, the conclusion based upon them must beconsistent with the results of the statistical study.

• In questions involving statistics, studies, or surveys, make sure to stay within thescope of the argument. Most of the wrong choices are simply outside the confinesof the situation.

Page 9: 10-PrepTest 10 Explsdl.keywin.org/1/e/1e1103afee7eee195fe5fbe511587657.pdfNo part of this book may be reproduced in any form, by photostat, microfilm, xerography or any other means,

KAPLAN LSAT PREP ________________________________________________________ LSAT Test X Explained: Section I

© K A P L A N 7

9. (D)The paleontologists’ conclusion that the centipedes lived on land is not supported bychoice (D), which is actually a neutral statement that has no bearing one way or theother on the argument. Knowing that the fossils previously thought to be the earliestland-dwelling animals weren’t found with the fossils of water-dwelling animals doesn’tsupport or damage the claim that the new fossils are indeed the earliest land-dwellers.

(A) If the legs were known to be of utility on land, they’re further evidence in supportof the conclusion that the centipedes lived on land.

(B) Provided nothing exists to differentiate the newly-found centipedes from theseland-dwelling ones, the fact that the previously-discovered centipedes were landdwellers can properly be used as support of the more recent finds.

(C) This statement would offer an explanation of how fossils of water-dwelling animalscame to be found in the rock. Such an explanation would further support the notionthat the newly-found centipedes were land-dwellers by clearing up the major groundsfor objection.

(E) The presence of fossils of other animals which could only breathe air — that is,animals that were not water based — lends support to the notion that the centipedesfound alongside them were land-based as well.

• Questions that call for you to identify the statement that provides the leastsupport for the argument really ask you to find the statement that either weakensor is neutral to the argument.

• When one choice “least supports” the argument, the remaining four mustsomehow strengthen the assertions made. Look for strengtheners and cross themoff.

• A statement that supports an argument need not go so far as to guarantee theaccuracy of the argument. It only has to make the argument more likely than itwould have been without it.

10. (E)The radio broadcaster claims that a responsibility to the “public interest” justifies theintrusions into the private lives of celebrities. When the broadcaster speaks of “publicinterest” in this way, he or she is talking about “for the public welfare or betterment.”However, the broadcaster then goes on to use the term “interest” in the context of “thepublic is interested/curious in this matter.” The broadcaster’s fallacy is in using the term“public interest” in two different ways.

(A) Quite the contrary, the broadcaster actually implies that when the public isinterested in a matter, no such right exists.

Page 10: 10-PrepTest 10 Explsdl.keywin.org/1/e/1e1103afee7eee195fe5fbe511587657.pdfNo part of this book may be reproduced in any form, by photostat, microfilm, xerography or any other means,

KAPLAN LSAT PREP ________________________________________________________ LSAT Test X Explained: Section I

8 © K A P L A N

(B) There’s no logical flaw committed by disregarding other grounds for criticism ofthe expose. The argument centers on the invasion of privacy issue; the flaw is in themanner of the broadcaster’s defense, not that he or she ignored other issues.

(C) Defining the term “excessively intrusive” isn’t necessary to the broadcaster’sargument. The term belongs to the critics, and the broadcaster was merely repeating it.

(D) In that legality is not discussed and morality is only vaguely hinted at in the critics’argument, there’s no confusion of these types of responsibilities/obligations.

• Some of the trickiest logical flaws center around the use of a word or phrase intwo different ways. When you know there’s a flaw in an argument, but you can’tput your finger on it, check to see whether a word or phrase that’s repeated in thestimulus is being used in a consistent way. If not, flag this as the logical flaw.

• Remember that flaws in the argument’s substance are different from flaws in themethod of argumentation. Read the question stem carefully to see which is beingasked.

• Keep the characters straight; choice (C), while not a strong flaw, is still part of thecritics’ argument, not the broadcaster’s.

11. (C)After stating that anyone in the vicinity of the fire would have seen it, the speaker placesThomas in the area and states that “he must have seen it.” All statements regardingThomas build up to this point; nothing more, nothing less. Therefore, “he must haveseen [the fire]” is this argument’s conclusion.

(A), (D), and (E) represent the logical link between the fact that Thomas went to thelibrary and the conclusion that Thomas must have seen the fire. As such, they’re part ofthe logical progression that eventually culminates in the conclusion just stated, not theconclusion itself.

(B) Thomas’ claim that he did not see the fire is specifically what the author is trying torefute in order to form the real conclusion: that he did.

• Remember that an argument’s conclusion need not come at the end of thestimulus; it’s often in the beginning, and sometimes will be found in the middle.

• The conclusion is the terminal point of the logical chain. For every statementmade in an argument, determine whether it represents the “final step” or whetherother statements are logical consequences of it.

• Statements of fact introduced into an argument are usually pieces of evidence,rather than conclusions.

Page 11: 10-PrepTest 10 Explsdl.keywin.org/1/e/1e1103afee7eee195fe5fbe511587657.pdfNo part of this book may be reproduced in any form, by photostat, microfilm, xerography or any other means,

KAPLAN LSAT PREP ________________________________________________________ LSAT Test X Explained: Section I

© K A P L A N 9

12. (B)According to the speaker, being near the fire was sufficient to guarantee seeing it, andsince Thomas met this sufficiency, he must therefore have seen the fire. (B) describes theargument similarly; if A is enough to ensure the occurrence of B, then if A happened, Bmust have happened as well.

(A) The speaker didn’t offer multiple pieces of stand-alone evidence. Rather, we’regiven a step-by-step buildup of sufficiencies.

(C) The only statement by another person that was argued against here was Thomas’assertion that he didn’t see the fire. However, the argument against that assertion was anillustration of how the facts don’t support his claim. His denial was never argued to be“tempting.”

(D) This argument was based on evidence derived from a specific incident only. Nogeneralizations from the past were used here.

(E) The speaker never asserted first-hand experience with respect to seeing the fire. Nosuch experience was used to form any general conclusions.

• The testmakers test formal logic by burying formal statements in the context of acasual argument. Be on the lookout for sentences that you can boil down to hardand fast rules.

• In questions that ask you to recognize a logical technique, attempt to describe itin your own words before approaching the answer choices. It’s often easier tosearch for the choice that matches your prediction rather than to blindly wadethrough all five.

• Make sure you’re clear on the concepts of necessary and sufficient: If A isnecessary for B to occur, B cannot happen without A. However, A alone may notbe enough to ensure that B happens: It’s necessary, but it’s not a guarantee. If, onthe other hand, A is sufficient for B, then the presence of A ensures that B willhappen; this is a guarantee. If B doesn’t occur, then neither does A.

13. (B)All this editorial states is that the president must have been acting in the nation’s bestinterests because he or she took an unpopular position. (B) points out the most glaringfallacy, i.e. that courageous action may nevertheless be the wrong action.

(A) Rather than confuse two personal qualities, as this choice suggests, the editorialmuddies the water between a personal quality (courage) and the quality of the actualdecision made.

(C) The portion of the editorial that speaks to citizen interest was directed at all citizenswho hold a particular position. Whether the number of citizens in that group is aminority or majority doesn’t have an impact on this argument.

Page 12: 10-PrepTest 10 Explsdl.keywin.org/1/e/1e1103afee7eee195fe5fbe511587657.pdfNo part of this book may be reproduced in any form, by photostat, microfilm, xerography or any other means,

KAPLAN LSAT PREP ________________________________________________________ LSAT Test X Explained: Section I

10 © K A P L A N

(D) If this were true, it would only reinforce the notion of the president’s courage. Itdoesn’t, however, describe the erroneous reasoning.

(E) This is way too broad. No assumption with respect to “any plan proposed by aparliament” is made; the argument speaks only to one particular plan.

• Read critically — always try to put the author’s ideas into some real-life context.Would you accept this reasoning if someone were trying to convince you of it, orwould you say “huh? What exactly is the connection between courage and actingin the nation’s best interest”?

• When testing an argument flaw, question whether fixing the “flaw” will repair theargument. If there’s still a hole in the reasoning, then look for a different choice.

14. (E)During inflation, consumers initially increase their spending but, as time drags on, theyeventually begin to cut back. The inconsistency between what consumers do at thebeginning of inflation and what they do when inflation continues over time can beexplained by looking at the patterns of change during the process. (E) offers a plausibleexplanation; if consumers’ overall spending power decreases as inflationary periodsprogress, it’s no wonder that they begin to curb their expenditures.

(A) conflicts with the evidence regarding consumer behavior at the beginning of aninflationary period. As for the protracted periods of inflation, while that might serve toexplain cutbacks in some areas of consumer spending, it’s nevertheless incomplete inthat it fails to explain why cutbacks also affect “routine” purchases.

(B) We’re specifically told that consumer behavior changes once the public becomesaware of inflation, so noting that such a time lag exists doesn’t affect the argument.

(C) We’re not concerned with whether a particular situation conforms to ageneralization. Rather, we’re comparing the differences in spending in two differentspecific situations, i.e. at the start of inflation and during its later stage.

(D) This may explain how the cycle may progress into a new non-inflationary phase,but doesn’t explain the odd consumer behavior during the protracted inflationary stagein question.

• Resolving discrepancies in “before and after” situations is often a matter offocusing on the intervening “during” time. What has occurred between then andnow? Why are things different? Look for choices that attempt to explain gaps intime.

Page 13: 10-PrepTest 10 Explsdl.keywin.org/1/e/1e1103afee7eee195fe5fbe511587657.pdfNo part of this book may be reproduced in any form, by photostat, microfilm, xerography or any other means,

KAPLAN LSAT PREP ________________________________________________________ LSAT Test X Explained: Section I

© K A P L A N 11

15. (A)The new theory is that asteroids are irrelevant; it was massive volcanic activities thatkilled off the dinosaurs. The necessary assumption being made is that the killer asteroiddidn’t cause the volcanic activity to occur, because if the asteroid were the causal factor,then the claim that the asteroid is irrelevant would fall apart.

(B) suggests that an asteroid collision did cause the extinction of dinosaurs, which isinconsistent with the new theory.

(C) The argument never addresses, nor is it dependent on, the relative amounts of timethat it would take for each form of catastrophe to bring about the dinosaur’s demise.

(D) Maybe there were other volcanic eruptions, maybe there weren’t. Either way, thisinformation has no real impact on the argument.

(E) Such knowledge is not critical to the argument. Asteroid or not, the author isconvinced that the volcanic activity, whenever it occurred, was responsible for theextinction.

• Analyze choices in “assumption” questions by asking: “If this statement werefalse, could the argument still hold”? If negating an answer choice blows up thewhole argument, then the choice is most likely a good assumption. (This wasmentioned earlier as the Denial Test.)

• When an author argues that a particular result was caused by one event and notanother, check to see if there can be any connection between the two events. If so,this could help answer a “flaw” question (there’s a connection that the authordoesn’t consider) or an assumption question (author assumes there’s noconnection).

16. (E)Two clear ways could be used to refute the asteroid-impact theory. One would be toshow that such an impact never occurred, and the other would be to establish that hadit occurred, dinosaurs lived beyond the impact. Choice (E) involves this secondmethod. If dinosaur remains were found above the layer of dust attributed to theasteroid, it can be inferred that dinosaurs were in existence after the time of the impact.

(A) and (C) don’t impact on the asteroid theory, as neither one mentions the asteroid orany other relevant facts surrounding extinction.

(B) suggests that the dinosaurs could have escaped local disasters but not globalcalamity, which is exactly what resulted from the asteroid impact. This, therefore, doesnothing to weaken this theory.

(D) suggests that some dinosaurs were able to live in very dry climates, but once again,this bears no relation to the asteroid nor to any relevant facts surrounding thedinosaur’s extinction.

Page 14: 10-PrepTest 10 Explsdl.keywin.org/1/e/1e1103afee7eee195fe5fbe511587657.pdfNo part of this book may be reproduced in any form, by photostat, microfilm, xerography or any other means,

KAPLAN LSAT PREP ________________________________________________________ LSAT Test X Explained: Section I

12 © K A P L A N

• Very often, the choice that weakens a theory is the one that negates or overrides acritical element in the logical chain.

• In testing possible argument weakeners, ask yourself, “Is it possible for thisstatement and the conclusion to exist in unison”? If the answer is yes, findanother choice.

17. (B)The author’s definition of a contract implies that there has to be an exchange of benefitsbetween two willingly engaged parties. The fact that the artist received “public funds,”however, isn’t the same thing as saying that the artist and the public (i.e. the actualpeople who will benefit from the artist’s work) came to such an agreement. Thisexample only partially subscribes to the definition of contract: One party, the artist,engages in the contract, but just because “public funds” are involved doesn’t mean thatthe “public” at large agrees to or even knows about this contract.

(A) The conferral of benefits on all parties wasn’t mentioned as a justification for acontract; rather, it was discussed as part of the definition of a contract.

(C) No; according to the author, whether a contractual obligation exists is not anabstract issue but rather an objective one, so speaking in abstraction wasn’t the errormade here.

(D) doesn’t deal with the argument at all, since there’s no discussion of mental oremotional activity in the passage; everything discussed is based on actual activity andobligation.

(E) At no point is it suggested that any issue is a matter of opinion. The passage impliesthat the artist’s obligation is a matter of fact, not of opinion. “Political processes” is alsoway off the mark.

• Questions that deal with law-related subjects don’t require any specializedknowledge of the law. The underlying reasoning is the crucial aspect, so treatthem the same as any other question on any other topic.

• In “method of argument” questions, it helps to have a simple restatement of theentire argumentative structure in your mind before scanning the choices.

• Unless an argument includes heavy-handed political references, avoid choicesthat deal with the argument on a political level.

Page 15: 10-PrepTest 10 Explsdl.keywin.org/1/e/1e1103afee7eee195fe5fbe511587657.pdfNo part of this book may be reproduced in any form, by photostat, microfilm, xerography or any other means,

KAPLAN LSAT PREP ________________________________________________________ LSAT Test X Explained: Section I

© K A P L A N 13

18. (E)One claim in the passage is that everyone will sometimes act in ways that areconsequential to the treatment they received while an infant. Since the treatment wasbeyond that person’s control, so are the consequential acts. Therefore, all adults willsometimes do things (relating to when they were infants) for which they should not beheld morally responsible. If everyone can disavow themselves of responsibility of atleast one action, then there is nobody who can be held responsible for all of his actions.

(A) Infants may perform actions that are not the consequence of treatment received. Inthose cases, the argument is silent as to responsibility.

(B) Similar to (A), this choice discusses the responsibility for actions that aren’tnecessarily out of the individual’s control.

(C) The argument doesn’t necessarily support this point, stating that in some cases,determining adult control over treatment received is difficult. Since nothing guides usas to when individuals who claim to lack control must take responsibility, this choicecan’t be fully resolved.

(D) The argument is limited to discussing responsibility for actions beyond theperson’s control. Nothing is said with respect to actions within control.

• Recognize when seemingly complex passages are actually a string of formalstatements: No one is morally responsible for things over which they have nocontrol. Infants are not morally responsible for treatment they receive. Everyonesometimes acts in ways that result from their treatment as infants. Try to breakthe argument down into its core formal logic components.

• Get familiar with the most common Logical Reasoning question types, andrecognize them even when the stem seems to go off into unfamiliar territory. Ifyou cut away all the fancy wording, this one simply asks for an inference basedon the stimulus.

19. (D)The issue here isn’t an increase in local taxes. The councillors simply want todiscontinue using city taxes to subsidize the bus system, and make up for the lostmoney by raising the bus fare. This, by itself, doesn’t imply that city taxes will go up, sothe fact that city voters oppose a potential tax hike (big surprise there) has no bearing onthe argument.

(A) The upshot of this is that by withdrawing the subsidies and raising the fare, thecity’s taxpayers would ultimately suffer, not benefit.

(B) Once again, the long term result is that the taxpayers are worse off — there’s morepollution and wear and tear on the roads — and the kicker is that taxes may even needto be increased in order to combat the new problems.

Page 16: 10-PrepTest 10 Explsdl.keywin.org/1/e/1e1103afee7eee195fe5fbe511587657.pdfNo part of this book may be reproduced in any form, by photostat, microfilm, xerography or any other means,

KAPLAN LSAT PREP ________________________________________________________ LSAT Test X Explained: Section I

14 © K A P L A N

(C) That the councillors agree that providing benefits to the low income residents isappropriate, is in sharp contrast to the proposition that only the taxpayers shouldreceive benefits from their contributions.

(E) By non-residents paying a wage tax, the argument can be made that the non-residents who ride the bus are, nevertheless, entitled to receive city benefits, as theyqualify as city taxpayers.

• When looking for a choice that doesn’t weaken an argument, remember that youdon’t necessarily have to find one that strengthens it; more likely, the rightanswer will be neutral or irrelevant to the whole thing.

• As in Logic Games, be careful to read the question stem carefully, and alwaystake note of the word EXCEPT.

20. (D)The conclusion, signaled by “therefore,” is that to the author, a certain type of activitydoes not qualify as “censorship.” If you scan the choices for their conclusions, only(D)’s fits that description: To (D)’s author, a certain type of activity does not qualify as“heroic.” Can it be that simple? Yep. You needn’t go further in your analysis andrecognize the fuller structure, when you know that only one choice is possible. For therecord, note that (D) and the stimulus share the same logical fallacy — assuming that anactivity sufficient for a definition (lack of communication = censorship; risk of life foranother’s benefit = heroism) is at the same time necessary for that definition.

(A) This choice properly deduces the contrapositive from an if/then statement: “If anaction causes unnecessary harm, then it’s unjust; since a just action causes harm, it mustbe necessary.”

(B) Whereas in the initial argument there’s a confusion between necessity andsufficiency, this choice makes the proper distinction. It shows that a condition necessaryfor a result (determination of good manners) is not necessarily sufficient for that result.

(C) The flaw in this argument is the assumption that two things that have one propertyin common (their name) must, therefore, have others in common as well. Thisassumption is not necessarily true. While a flawed argument, it is a different flaw fromthe one which we are seeking.

(E) This argument defines beauty as a subjective term and argues against use of theterm since different people will reach different conclusions. Again, this structure isdifferent from the argument for which we need to find a parallel.

• In parallel logic questions, restate the conclusion in abstract terms and compareeach choice against this restatement. If — as often happens — only one conclusionmatches the original, then you’ve found the answer. Period.

Page 17: 10-PrepTest 10 Explsdl.keywin.org/1/e/1e1103afee7eee195fe5fbe511587657.pdfNo part of this book may be reproduced in any form, by photostat, microfilm, xerography or any other means,

KAPLAN LSAT PREP ________________________________________________________ LSAT Test X Explained: Section I

© K A P L A N 15

• Statements that are logically parallel need not have all logical elements in thesame sequence. Provided all elements of the first argument exist in the second,even in a different order, the two arguments are parallel.

• When parallel argument questions also ask you to match logical flaws, rememberthat a choice with a flaw different from that of the original argument is a wrongchoice. Not only must the structure be the same, but so must be the flaw.

21. (B)The evidence offered in this argument is that English poets use a lax definition of a termthat has a more specific meaning in Japanese culture. The conclusion, however, is muchbroader in that it accuses the poets of showing no respect for foreign traditions. The leapfrom not adhering to one small facet of one culture to showing no respect for any facetof any foreign culture is the clear logical flaw.

(A) There’s no confusion of fact and feeling in this argument. The conclusion is rootedin fact; a haiku is defined in a particular way, yet English poets use the term to describepoetry not conforming to this strict definition.

(C) On the contrary; the author certainly does present evidence: This is a haiku, and thisis what the English try to pass off as a haiku. These aren’t stereotypes; they’re facts.

(D) The argument doesn’t overlook this possibility. In fact, in order for such a broadconclusion to be reached, the author must in fact rely on the idea that the case cited isnot unique.

(E) has no bearing on the argument, as it’s completely beyond the scope.

• Reading the question stem before the stimulus is a good strategy for all LogicalReasoning questions, but is even more applicable to “flaw” and “weaken”questions: Your job is easier when you know in advance that there’s somethingwrong with the reasoning in the stimulus.

22. (C)The argument is based on a link between dreaming and memory, so the correct choiceshould make reference to this link. The hypothesis suggests that dreaming serves to“unclutter” the mind, thus “freeing up memory.” It would follow that when dreamingis interrupted, so too is this process of erasing our brain’s meaningless connections,which would in turn negatively impact on memory space.

(A) Not necessarily; maybe these dimbulbs have the fewest parasitic connections todispose of, and therefore need very little time for sleep. Besides, only dreaming time isdiscussed, not sleeping time.

(B) The hypothesis implies that, after dreaming, a person’s memory will contain fewer“accidental associations.” However, there’s no indication that the “erasure” processeliminates virtually all meaningless associations.

Page 18: 10-PrepTest 10 Explsdl.keywin.org/1/e/1e1103afee7eee195fe5fbe511587657.pdfNo part of this book may be reproduced in any form, by photostat, microfilm, xerography or any other means,

KAPLAN LSAT PREP ________________________________________________________ LSAT Test X Explained: Section I

16 © K A P L A N

(D) Nothing in the hypothesis implies that we dream only in terms of parasiticconnections. There’s no reason why a person’s dreams couldn’t contain meaningfulassociations that he or she accurately describes upon waking.

(E) The spiny anteater is described as the only mammal that does not have rapid eyemovement sleep. No mention is made of non-mammals in this argument. Also, there’sambiguity relating to the difference between “REM sleep” and “dreaming”: Humanstypically experience the latter during the former, but we can’t be sure that this appliesto “all animals.”

• A statement that is “strongly supported” by an argument is generally either alogical “next step” or, quite typically, a restatement of the argument’s crux.

• When an argument is based upon establishing a relationship between two thingsor events, look for choices that focus on both of the key aspects and the linkbetween them.

23. (D)The example of the spiny anteater is used to demonstrate the idea of an inverserelationship between brain size and dream activity, a phenomenon that fits very nicelyinto the parasitic-connection hypothesis. While it certainly doesn’t prove thehypothesis, the fact that the spiny anteater doesn’t dream and has such a large brainrelative to its overall size can be explained by a prediction derived from the hypothesis(extra memory space is needed for animals that don’t dream), and therefore serves topartially validate the theory.

(A) Facts about one species (the spiny anteater) provide confirmation for hypothesesabout species that were different in a major relevant respect, i.e., brain size.

(B) is off the mark: Only one prediction is made and we can’t go so far as to say that it’s“true,” only that it’s supported by a specific example.

(C) For a very wordy principle, this complicated choice hardly says anything at all thatseems related to the argument.

(E) There are no “competing hypotheses,” so right away we can see that this principledoesn’t apply.

• “Principle” questions involve fitting a specific situation into a global generality(or, occasionally, vice versa).

• The answer to principle questions is usually the choice that expresses the keyconcepts and contains the key terms that the other choices leave out. Notice thesimilarity between “this fact provides some confirmation” in the stimulus and “ahypothesis is partially confirmed” in the correct answer.

Page 19: 10-PrepTest 10 Explsdl.keywin.org/1/e/1e1103afee7eee195fe5fbe511587657.pdfNo part of this book may be reproduced in any form, by photostat, microfilm, xerography or any other means,

KAPLAN LSAT PREP ________________________________________________________ LSAT Test X Explained: Section I

© K A P L A N 17

• When choices employ abstractions (“facts,” “an action or an object”), try to beconscientious in relating back to the specifics of the stimulus. You can’t evaluatesuch choices until you understand the general framework in the context of thepassage.

24. (E)From the time that antibodies appear (a week into the virus), up through the next year,the antibody test can be used to determine how long a person has had the virus.However, if the test were to be administered during the first week of the virus, beforeantibodies were produced, the test would not detect the virus.

(A) All we know is that the antibodies are known to increase for about a year. Nothingin the argument indicates that this increase stops when the virus is killed.

(B) We’re told that the antibody test can be used to establish whether someone has thevirus; however, that doesn’t exclude the possibility that other detection methods alsoexist — for example, maybe virus X is characterized by unmistakable outward signs.

(C) is way too broad to be supported by this stimulus, which deals only with antibodiesrelated to virus X. For all we know, antibodies in general can be produced for a muchwider range of cases than the one mentioned in (C).

(D) We’re given no information about what happens after one year — perhaps there is acontinued increase, or perhaps a leveling off. Since we don’t know, we can’t infer thisconclusion.

• When an author introduces a time frame, take note of it. The question willprobably center around it.

• When applicable, try to visualize the events on a time line. If the argumentdiscusses what happens between time A and time B, question what occurs beforeA and what comes after B. If you can’t resolve this with certainty, at least makenote of the ambiguity.

• If the argument deals with a fixed time frame, be extra wary of choices that dealwith an indefinite future.

• Be careful not to answer according to your own knowledge of a situation. Forexample, perhaps you learned in a biology class that it’s not unusual for someantibodies to stop increasing in number once the virus has been defeated;nonetheless, there’s nothing in this particular passage that supports this.

Page 20: 10-PrepTest 10 Explsdl.keywin.org/1/e/1e1103afee7eee195fe5fbe511587657.pdfNo part of this book may be reproduced in any form, by photostat, microfilm, xerography or any other means,

KAPLAN LSAT PREP ________________________________________________________ LSAT Test X Explained: Section I

18 © K A P L A N

25. (E)Here’s the argument, restated in variables: A problem with X (wealth) is bad for Y(democracy), since X is necessary for Z (political power), and Y (democracy) dependsupon Z (political power). The same structure is found in choice (E): A problem with X(leisure time) is bad for Y (health), since X (leisure time) is necessary for Z (regularexercise), and Y (health) depends upon Z (regular exercise).

(A) X and Y are a bad combination, since together they lead to Z.

(B) If X, then Y, since Z.

(C) It’s hard to do X with characteristic Y, since Y depends on Z, whereas X does not.

(D) X is important for Y, but since X can sometimes cause Z, the effect of X on Y isuncertain. The original argument did not have this element of uncertainty and, on thatpoint alone, is not parallel to this choice.

• Parallel Reasoning involves mimicking the structure of an argument. Try not toget caught up specifics; it’s the general framework of the original that you’relooking to locate in one of the answer choices.

• Some parallel reasoning stimuli lend themselves well to algebraic representation.If you can condense the structure to letters, do so, and then look for the choicewith the same exact structural breakdown.

• Stay within the tone of the original — if its terms are stated in the absolute, usingwords such as “always,” stay away from choices that use words like “sometimes”or “usually.”

• As a shortcut, notice that the conclusion of the stimulus says that a certaincondition always causes a particular result, and only (E)’s conclusion doeslikewise. (The choice doesn’t use the word “always,” but it’s implied.) Your searchfor the right answer could have begun and ended right there!

Page 21: 10-PrepTest 10 Explsdl.keywin.org/1/e/1e1103afee7eee195fe5fbe511587657.pdfNo part of this book may be reproduced in any form, by photostat, microfilm, xerography or any other means,

© K A P L A N 19

SECTION II:LOGIC GAMES

Page 22: 10-PrepTest 10 Explsdl.keywin.org/1/e/1e1103afee7eee195fe5fbe511587657.pdfNo part of this book may be reproduced in any form, by photostat, microfilm, xerography or any other means,

KAPLAN LSAT PREP _______________________________________________________ LSAT Test X Explained: Section II

20 © K A P L A N

GAME 1 — Nine Students, Three Classes(Q. 1-5)

The Action: Distributing nine students into three classes, three to a class, based on testscores — the top three in class 1, the middle three in class 2, and the bottom three inclass 3. The Key Issues that will be tested for in the questions involve typical groupingconcerns:

1) Which class is each student placed in?

2) Which students can or must be in the same class?

3) Which students cannot be in the same class?

The Initial Setup: Like any other grouping game of distribution, it may be helpful tocreate a sketch containing the groups that the entities are to be broken into, in this case,the 3 classes:

F G H I J K L M N

Cl. 1__ __ __

(Top)

Cl. 2__ __ __

(Middle)

Cl. 3__ __ __

(Bottom)

There’s nothing to fill in until we hit the rules, so they’re up next.

The Rules: Here’s where this game gets interesting. The rules appear to be sequencingrules, rather than the typical grouping rules we would expect to accompany theopening scenario. In a sequence game, you would be asked to deduce from these rulesthe students’ scores relative to one another. (For example, combining Rules 1 and 2, youcould deduce that Ida scores higher than Kathy.) While this type of reasoning willcertainly be called upon, the questions themselves stick to the grouping aspect of thegame: You need to use the information on the students’ relative scores (sequencing) tobreak up the students into three classes of three students each (grouping). Thus, whatwe really have here is a “hybrid” game.

Page 23: 10-PrepTest 10 Explsdl.keywin.org/1/e/1e1103afee7eee195fe5fbe511587657.pdfNo part of this book may be reproduced in any form, by photostat, microfilm, xerography or any other means,

KAPLAN LSAT PREP _______________________________________________________ LSAT Test X Explained: Section II

© K A P L A N 21

The rules are laid out in perfect order in a sequence of decreasing scores. Here’s a wayto get all of the information into one compact sketch:

IG

J KM F L

HN

Key Deductions: Every score is below Ida’s, and every score other than Ida’s is belowGlen’s. It should be clear, then, that I has the top score, and G the second highest.Therefore, I and G have to be in class 1—that’s a certainty! The only students thatcould possibly have the third highest score are Jan and Kathy. Therefore, rounding outclass 1 must be either J or K (which, incidentally, answers question 1). While Nickappears to be at the bottom of the left branch of the “tree”—and here’s where one’s ownsketch can be misleading—remember that the scores of K, F, and L bear no directrelation to his. It’s possible therefore that K, F, and L could all be placed in class 3, whileNick is placed in class 2. In any case, Ida and Glen are the only students whose exactplacement can be determined at the point the questions begin.

The Final Visualization: Along with the sketch of the rules, here’s what we’re armedwith going into the questions:

F G H I J K L M N

Cl. 1__ __ __

Cl. 2__ __ __

Cl. 3__ __ __I G

J/K

The Big Picture:

• Be prepared for games that require a combination of skills. “Hybrid” games havebecome quite popular on the LSAT.

• A “free-floating” or indefinite sequence is one in which the most important thingis each entities’ position relative to the other entities. In these games, you need topay careful attention to the number of entities that must be above or below theothers. Doing so often leads to Key Deductions.

Page 24: 10-PrepTest 10 Explsdl.keywin.org/1/e/1e1103afee7eee195fe5fbe511587657.pdfNo part of this book may be reproduced in any form, by photostat, microfilm, xerography or any other means,

KAPLAN LSAT PREP _______________________________________________________ LSAT Test X Explained: Section II

22 © K A P L A N

• Make sure that you recognize the difference between hard and fast relationships(e.g. Ida scoring higher than Kathy, by way of Rules 1 and 2) and undeterminedrelationships (e.g. Fred and Laura; they both score lower than Kathy, but we can’ttell how they scored relative to each other).

• The most important role of shorthand is to make the game’s informationaccessible so that you can fully understand the meaning of the rules at a glance. Ifyour own sketch misleads you, it’s a sure sign that you’re not thinking enough inthe working-out of the game. Your sketch has to reflect your thinking, nothingmore and nothing less.

The Questions:

1. (B)We virtually answered this one while formulating the Key Deduction: I and G arefixtures of class 1, while the final spot will go to either J or K, for two possiblecombinations of students for this class.

• The first question often helps reinforce the major concepts of the game—as thisone forces you to realize, if you hadn’t already done so, that I-G-J and I-G-K arethe only possibilities for class 1.

2. (C)We know from Q. 1 that Kathy could join Ida and Glen in class 1, leaving Jan for class 2.But one glance at your sketch should tell you that it’s impossible for Jan to be placed inclass 3, because there are three students’ scores (M, H, and N) that are necessarily lowerthan Jan’s. Even if the scores of K, F, and L were higher than Jan’s, it still wouldn’t beenough to force her down into class 3.

(A), (D), (E) Fred, Kathy, and Nick could all wind up in class 2 or class 3.

(B) As we know by now, Glen must be in class 1, and therefore can’t be in 2 or 3.

• Once armed with the kind of scratchwork employed in this game, don’t assumethat you have to do a lot of writing on your page in order to answer questions.Unnecessary extra drawing is a hindrance, not a help. Merely consulting yoursketch and mentally manipulating the entities will usually suffice.

3. (C)The first student you should cross off is Jan; we just saw a second ago in Q. 2 that shecan’t be in class 3. With (B) gone, our work in Q.1 helps fill in the rest of the picture:The only student besides Jan who can join Ida and Glen in the top class is Kathy. At thispoint, there’s no need to even check whether she can be in the other two classes; she’s theonly student among the remaining choices eligible for class 1, and that’s enoughevidence to circle (C) and move on.

Page 25: 10-PrepTest 10 Explsdl.keywin.org/1/e/1e1103afee7eee195fe5fbe511587657.pdfNo part of this book may be reproduced in any form, by photostat, microfilm, xerography or any other means,

KAPLAN LSAT PREP _______________________________________________________ LSAT Test X Explained: Section II

© K A P L A N 23

• The questions don’t exist in a vacuum: Use what you learn in previous questionsto help you pick winners or eliminate choices in the current question.

4. (C)The way to test a choice here is to place its two students into class 2 (mentally, or bydrawing a little freehand sketch) and then see if any ambiguity still exists. If there’smore than one way to place the rest of the students, then that choice is wrong. If Hillaryand Jan, choice (C), are in class 2, then Mike, whose score comes between theirs, must bein class 2 also. Furthermore, with Jan in 2, Kathy is the only eligible student to join Idaand Glen in class 1. This leaves underachievers Nick, Fred, and Laura in the basementin class 3. So by placing Hillary and Jan in class 2, we accomplish the desired result.

(A) leaves the placement of Mike and Laura unresolved; one will be in 2, the other in 3,but we can’t tell which goes where.

(B), (E) Neither determines whether J or K joins I and G in class 1.

(D) leaves Fred and Mike to float between classes 2 and 3.

• “Backwards reasoning” questions are those that ask you to supply a piece ofinformation that will lead to a particular result. Unless an intuitive solutionjumps out at you, these usually involve simply trying out the choices. If you’restrapped for time, you may wish to skip this type, moving instead to questionsthat give you a more concrete starting point.

5. (E)Laura and Mike is the pair that can’t join Fred, because creating this threesome wouldleave too few students (just H and N) for the bottom class, and too many (I, G, J, and K)for the top class.

(A) Add Fred, stir, and you have an acceptable class 3.

(B) Send K to the head of the class, M, H, and N to the basement, and these two andFred end up in class 2.

(C) Kathy, Laura, and Fred can share class 2 or class 3 together.

(D) These two and Fred can comprise class 2, if K goes to class 1 and L, H, and N fall toclass 3.

Page 26: 10-PrepTest 10 Explsdl.keywin.org/1/e/1e1103afee7eee195fe5fbe511587657.pdfNo part of this book may be reproduced in any form, by photostat, microfilm, xerography or any other means,

KAPLAN LSAT PREP _______________________________________________________ LSAT Test X Explained: Section II

24 © K A P L A N

GAME 2 — Movie Reviews(Q. 6-12)

The Action: The game may appear complex, but the action is actually very simple:matching up reviewers to movies. Furthermore, there’s really only one Key Issue,which will form the basis for every question:

1) Who reviews which movie?

The Initial Setup: Creating a list works very well for keeping track of information inone-to-one matching games such as this. Try listing the reviewers on one line, leavingspace underneath to write in the movie each one reviews, like so:

F G HMoviesM RS W

J K L

(Of course you could also make 4 columns, one for each movie, and list the peoplebeneath them as you learn about the assignments.)

The Rules:

1) In terms of our sketch, this rule states that there must be exactly one movie letter (M,R, S, or W) underneath each reviewer, and that no movie may be left out of the pictureentirely.

2) You can draw an arrow connecting H and F in your sketch to indicate that these tworeview the same movie.

3) We can combine this with the previous rule to deduce that Lena’s movie can’t be thesame as Hilda’s and Frank’s. Whichever movie Lena reviews, exactly one other personreviews it, and no one else.

4) The most concrete rule — put it right into the sketch.

5) “M/W” over J in your sketch should suffice to remind you that Jackie must reviewMystery or Wolves.

6) If Wolves is off limits to Hilda, then it’s off limits to Frank as well, thanks to Rule 2.Indicate this and we’re ready to piece this thing together.

Page 27: 10-PrepTest 10 Explsdl.keywin.org/1/e/1e1103afee7eee195fe5fbe511587657.pdfNo part of this book may be reproduced in any form, by photostat, microfilm, xerography or any other means,

KAPLAN LSAT PREP _______________________________________________________ LSAT Test X Explained: Section II

© K A P L A N 25

Key Deductions: Jackie has a choice between M and W. If she chooses M, then she andGeorge both review M. Meanwhile, Frank and Hilda must review the same movie; let’ssay they review R—then this would leave Karl and Lena, and two movies, S and W, yetto be reviewed. Can this be? No!: Rule 3 forces Lena to review the same movie asexactly one other person, and in this case, that would have to be Karl — but then thisway, only 3 of the 4 movies get reviewed, which violates Rule 1. So, Jackie cannotreview M (which started this whole line of deduction), and therefore she must reviewW. Meanwhile, we can see that if Frank and Hilda were to review M along with George,the same thing would happen. Therefore, Frank and Hilda must review either Retreator Seasonings.

The Final Visualization: Having done all of this good work, we’re ready to tackle thequestions with this information at our disposal:

F G HM W

J K L

R or SSame as just oneother person

By the way, Seasonings is an actual movie, written by Waldo Salt and directed by HerbRoss, starring Rosemary Harris and Barbara Pepper.

The Big Picture:

• Perhaps working out the analysis above would take a precious 2 or 3 minutes ofyour time on Test Day. It’s well worth it. The questions fall so much faster andwith so much more ease when you’re armed with the powerful deductions thatresult from this thinking stage.

• Remember, you don’t have to work on this game early on just because it’s printedsecond in the booklet! If you get stuck, or if you see that the game requires orbenefits from careful work and extra time, save the game till later! Earn that timeby working more efficiently on the other games.

• When one entity has a choice of only two outcomes, it’s worth your while toinvestigate both. Usually, this helps break the game down into two distinctparadigms, and the questions will dictate which one is relevant at the moment. Inthis case, however, our detective work determined that one of the options isimpossible altogether, leading to our Key Deduction.

• Make sure you recognize the distinction between “exactly” and “at least.” Don’tjust skim over these words; they’re very important factors in how games play out.

Page 28: 10-PrepTest 10 Explsdl.keywin.org/1/e/1e1103afee7eee195fe5fbe511587657.pdfNo part of this book may be reproduced in any form, by photostat, microfilm, xerography or any other means,

KAPLAN LSAT PREP _______________________________________________________ LSAT Test X Explained: Section II

26 © K A P L A N

The Questions:

6. (A)When we combined Rules 2 and 3, we saw that Lena must review a different movie fromHilda and Frank. One of the Key Deductions is that Hilda and Frank can only reviewRetreat or Seasonings. Putting these together with the if-clause stating that Lena reviewsSeasonings, we find that Hilda (and Frank) must review Retreat.

(B) directly contradicts Rule 5, not to mention the Key Deduction (that Jackie mustreview Wolves).

(C), (D), (E) In order to satisfy Rule 3, Karl must join Lena in reviewing Seasonings.

• When you’ve deduced up front as much as we have in this game, don’t besurprised when the first couple of questions are very simple. Some test-takersdoubt themselves at this point, but you should feel confident that if you’ve donethe right work up front, you’re entitled to some quick and easy points.

7. (E)If Karl doesn’t review Seasonings, then Lena can’t either (Rule 3). Well, someone has toreview this movie, so the task is left to Frank and Hilda. Is that the answer? Scanningthe list . . . yup, it is. Thanks to our earlier work, another point is ours in only a fewseconds.

(A), (B), (C) Lena could review Mystery with George, Retreat with Karl, or Wolves withJackie, so (A) and (B) could be true, but (C) is impossible, since F and H are reviewingSeasonings.

(D) No, we know that (E) is true; these two review Seasonings.

• Don’t go to the choices before you have to. When given hypothetical information,always work with it until you figure out something new. Then scan the choicesand see if your answer is there. If it’s not, then that means you can and must takethe chain of deductions at least one step further.

8. (E)Given the nature of the previous two questions, this one is a gift. We just got throughwith a situation in which F and H review Seasonings, and Lena can review anything butSeasonings — Mystery with George, Retreat with Karl, or Wolves with Jackie. Can Lenareview Seasonings under different circumstances? You bet — the hypothetical of Q. 6comes right out and tells us it’s possible. It’s therefore possible for Lena to review allfour movies.

• This is another example, similar to Game 1, Q.3, that illustrates how the questionsdon’t exist in a vacuum. Use what you learn in previous questions to help youpick winners or eliminate choices.

Page 29: 10-PrepTest 10 Explsdl.keywin.org/1/e/1e1103afee7eee195fe5fbe511587657.pdfNo part of this book may be reproduced in any form, by photostat, microfilm, xerography or any other means,

KAPLAN LSAT PREP _______________________________________________________ LSAT Test X Explained: Section II

© K A P L A N 27

• When a hypothetical situation is posed in an if-clause of a question stem, for thesake of later questions you can rest assured (unless it’s stated otherwise by a rulechange) that the situation posed is indeed possible.

9. (E)With no hypothetical information to go on, we have to rely on our Key Deductionsmore than ever. You should’ve used them to eliminate choices, and if you started with(E), you got the right answer right away. Lena (and Karl) can review Retreat, while Frank(and Hilda) review Seasonings.

(A), (B) Frank and Hilda must review either Retreat or Seasonings; this quickly kills (A)and (B), which have Frank reviewing Mystery and Wolves.

(C) is no good, as we deduced that Jackie must review Wolves.

(D) is impossible also, because this would leave only Frank and Hilda to review Retreatas well as Seasonings, but since they must review the same movie, one of these movieswould not get reviewed. This violates Rule 1.

• Even when you’re given no information to go on, and you’re therefore forced totest out choices, keep the Key Deductions in the forefront of your mind, andyou’ll be able to quickly scan the list and cross off bad choices withoutlaboriously working them out.

10. (B)Lena can’t review Mystery with Karl because George reviews Mystery (Rule 4), and themovie Lena reviews must be reviewed by exactly one other person (Rule 3).

(A), (E) are both possible, as long as Karl reviews Retreat or Seasonings, and theHilda/Frank team reviews whichever one Karl doesn’t.

(C), (D) Lena can see Retreat with Karl if Frank and Hilda review Seasonings, or she cansee Seasonings with Karl if Frank and Hilda review Retreat.

• Read question stems carefully, making sure not to miss the word EXCEPT whenit appears. If you have a tendency toward carlessns. . . carelsnenss. . . carelessness,you might benefit from quickly re-checking the question stem before markingyour choice, in order to ensure that you’re answering the question that’s asked.

Page 30: 10-PrepTest 10 Explsdl.keywin.org/1/e/1e1103afee7eee195fe5fbe511587657.pdfNo part of this book may be reproduced in any form, by photostat, microfilm, xerography or any other means,

KAPLAN LSAT PREP _______________________________________________________ LSAT Test X Explained: Section II

28 © K A P L A N

11. (C)This is like taking the gist of Rule 3 and applying it to Karl. Like Lena, thecircumstances dictate that Karl can’t review the same movie as Frank and Hilda.Furthermore, if he reviews Mystery with George, or Wolves with Jackie, then therewould be no way to maintain Rule 3 and still include every movie for review. Karlmust therefore review the same movie as Lena. So here’s what we have: George reviewsMystery, Jackie reviews Wolves (from the Key Deduction), the Frank/Hilda pair mustreview Retreat or Seasonings, and the Karl/Lena pair must review Retreat or Seasonings(whichever one Frank and Hilda don’t take), and that gives us the answer.

• Don’t be intimidated by a long question stem.

• When one entity is placed under the same restrictions as another, often times theimmediate results are the same, and recognizing this can save you work and time.

12. (B)(B) contains the acceptable lineup that we’ve seen quite a few times already.

(A) and (C) violate Rule 5.

(D) violates Rule 3.

(E) violates Rule 4.

• When facing an “acceptability” question, simply check the choices against eachrule, eliminating the ones that violate the rules until you’re left with only onechoice. If you do your work carefully, there’s no need to check the remainingchoice; just circle it and move on.

• Some students were a little thrown by the physical appearance of this questionon the page — centered across the columns and all. Actually, such questions tendto be “acceptability” questions and therefore quite simple. Remember that theway each game appears on a page is dictated by the content, not by a desire toconfuse or paralyze test takers!

Page 31: 10-PrepTest 10 Explsdl.keywin.org/1/e/1e1103afee7eee195fe5fbe511587657.pdfNo part of this book may be reproduced in any form, by photostat, microfilm, xerography or any other means,

KAPLAN LSAT PREP _______________________________________________________ LSAT Test X Explained: Section II

© K A P L A N 29

GAME 3 — The Word Game(Q. 13-18)

The Action: A genuine “oddball” game: We have to combine at least four letters toform words, and combine exactly five words to form a sentence. We’ll know more oncewe delve into the rules, but for now there appears to be two Key Issues:

1) What groups of letters form an acceptable word?

2) What groups of words form an acceptable sentence?

The Initial Setup: There’s really no way to “set this one up,” per se, in the sense of“how do I draw it?” Most of the work will involve looking at or forming words on thepage, and thinking about which words and sentences satisfy the rules and which onesdon’t. Each question will dictate what, if anything, could be jotted down on the page inorder to get the correct answer.

The Rules:

1) Self explanatory; just don’t miss the phrase “in alphabetical order.”

2) The first word is picked randomly, which means that any old combination of four ormore letters will do. There are three “operations” to keep track of; drop a letter, add aletter, and replace a letter with another. The second to the fifth words are formed byperforming one of these operations on the word to each one’s immediately left.

3) Translation: No more than three words can begin with the same letter.

4) This means that the operation that turns word 1 into word 2 must be different fromthe operation that turns word 2 into word 3, and so on. For example, let’s say that youadd a letter to word 1 to form word 2; you can’t then add a letter to word 2 to formword 3 — you must either drop one or replace one with another.

You might want to use your pencil to indicate or keep track of the changes betweenwords: + for add a letter, - for delete one, and perhaps “ch” or ø or some such toindicate a replacement.

Key Deductions: There’s not enough to go on to form any Key Deductions. You’llsimply have to tackle each question with the help of the rules as stated.

Page 32: 10-PrepTest 10 Explsdl.keywin.org/1/e/1e1103afee7eee195fe5fbe511587657.pdfNo part of this book may be reproduced in any form, by photostat, microfilm, xerography or any other means,

KAPLAN LSAT PREP _______________________________________________________ LSAT Test X Explained: Section II

30 © K A P L A N

The Big Picture:

• One way to latch onto the action of a game is to place the game in a real-lifecontext: How would you handle this if you were playing this word game withfriends? Maybe this is something you would never do in real life, but if youbreak down the mystique, and approach the games as ordinary everydaysituations, you may find you’re able to think through them more clearly.

• Break down long and complex-sounding rules into easy to manage concepts bytranslating them into your own words. If you’re unsure of a rule’s meaning, forman actual example, in your head or on your page, to help clarify yourunderstanding.

• Oddball games tend to involve rules that don’t seem to link up easily, whichpartly explains our inability to extract Key Deductions from this one. As a result,you can often answer questions in these types of games by focusing on one ortwo rules at a time; the skill involved is isolating the relevant rule for eachquestion.

• Always read critically — sometimes a phrase like “in alphabetical order,” fromRule 1, gets lost in the shuffle. Such mistakes inevitably come back to haunt uslater. To avoid this, try quickly reviewing the rules — just a couple of seconds —after each question.

• If nothing else, this game illustrates what we’ve been telling students for years:Don’t jump in asking “How do I draw it”? Let your sketch — if any — come outof your thinking about the game and its action!

The Questions:

13. (B)(A) bites the dust right away, because it contains a word with only three letters.Contrary to Rule 3, choice (E) has four words beginning with F. (C) and (D) violate Rule4 — they both make use of the replacement operation twice in a row: In (C), “dteam” to“gleam” to “glean,” and in (D), “reed” to “seed” to “seeg.” The remaining choice, (B),must therefore contain an acceptable sentence.

• When facing an “acceptability” question, simply check the choices against eachrule, eliminating the ones that violate the rules until you’re left with only onechoice. If you do your work carefully, there’s no need to check the remainingchoice; just circle it and move on.

• The first question can solidify your conception of a game. Answering this onehelps to reinforce the action of this somewhat odd game, and reassures us that wehave a good grasp of it.

Page 33: 10-PrepTest 10 Explsdl.keywin.org/1/e/1e1103afee7eee195fe5fbe511587657.pdfNo part of this book may be reproduced in any form, by photostat, microfilm, xerography or any other means,

KAPLAN LSAT PREP _______________________________________________________ LSAT Test X Explained: Section II

© K A P L A N 31

14. (D)This one hinges on Rules 1 and 3: The words must be in alphabetical order, and nomore than three can start with the same letter. Starting from the end, let’s test out Z first:Can Z be the first letter of the first word? No; the most we can have are three wordsbeginning with Z, and there’s nowhere to go but down from there. If the first wordbegins with Z, it’s inevitable that somewhere along the chain, the alphabetical order willbe broken. What about Y? This works; if you can’t visualize it, test out an example onyour page, such as: YABC, YABCD, YBCD, ZBCD, ZBCDX.

• Decode confusing or wordy question stems by paraphrasing, in your own words,what the question is after.

• When looking for the last letter of the alphabet that’s possible, we suggested thatyou test out the letter that’s closest to the end of the alphabet first. Why? Becauseif it worked, it would’ve been our answer. However, if we had tested “t” (choice A)and found it to be acceptable, we would still have to have tested “w,” and so on,until we found the last letter of the alphabet that fit the bill. The same holds truefor other games’ “maximums” and “minimums” questions; if a question seeks thegreatest possible number of something, try out the biggest choice first, and viceversa for minimums.

15. (C)This requires a straight application of the rules. Simply test each choice until you findthe word that can be derived from “blender” and turned into “sender.” If we simplydrop the “b” from word 1, forming “lender,” and then add an “s” to “lender” to form“slender,” we’re done. (A quicker, more intuitive way is to notice that “lender” is theonly choice that’s part of both word 1 and word 3.)

(A) We can go from “blender” to “bender” easy enough, but can’t go from “bender” to“slender,” as this would require replacing one letter with two letters.

(B), (D), and (E) are all impossible to derive from “blender,” as each would entailreplacing two letters with one.

• Even games that appear unusual have easy questions scattered throughout; Q’s.13 and 15 are simply testing your understanding of the rules.

16. (D)Here we’re looking for the fewest number of letters permissible for word 4 if the firstword has nine letters. Well, just work your way across: Since we’re trying to end upwith the fewest letters possible, let’s immediately drop one so that word 2 has eightletters. We want to continue to drop letters, but we have to hold off for at least the nextword — Rule 4 requires that a different operation be performed to form word 3. Well, wedon’t want to add a letter, that would go against our purpose. So let’s substitute a letter,so that word 3 still contains eight. Now we’re free to drop another letter, which gives usthe answer: a seven-letter word 4.

Page 34: 10-PrepTest 10 Explsdl.keywin.org/1/e/1e1103afee7eee195fe5fbe511587657.pdfNo part of this book may be reproduced in any form, by photostat, microfilm, xerography or any other means,

KAPLAN LSAT PREP _______________________________________________________ LSAT Test X Explained: Section II

32 © K A P L A N

• Remember that the main task in an oddball game is to weed out the aspect oraspects of the game that are relevant to each individual question. In this one, theletters themselves are irrelevant — the sole issue is the number of them in certainwords.

17. (E)Translation of stem: If “clean” is the first word, where in the order could “learn” showup? It can’t be second; we’d have to drop the “c” and add and “r,” which we can’t doall at once. Cross off any choice that includes “second,” namely, choices (A) and (D).“Learn” could be third, however, if we do exactly what we just said, but in two steps:clean, lean, learn, etc, etc. This narrows the choices down to (B), “third” and (E), “third,fourth, fifth.” At this point, a test of either fourth or fifth will reveal the answer. Nomatter which we test, if learn could be in that spot in the ordering, then (E) is correct. Ifit can’t, then (B)’s the one. If you try it, you’ll find that “learn” could be fourth: “clean,flean, lean, learn, xearn” is only one of many examples that shows that “learn” could befourth, proving that (E) is correct.

• Don’t be afraid to skip a question that appears wordier or more complex than theothers; return to it later if you have the time, or else just guess. It may turn out tobe easy after all — but there’d be no harm done.

• Employing the answer choice technique above, you’ll find that for a question likethis you don’t have to test out every position from 2 to 5. Testing out a few willnarrow the choices down in such a way that you’ll eventually be left with thecorrect answer, while saving on time.

18. (C)This question bears a remarkable resemblance to Q. 16 — both give the number ofletters in the first word, and then ask about the number of letters in another word. So, ifthe first word has four letters, what’s the maximum number of letters the fifth wordcould have? Let’s go about it the same way we handled 16 — work down the line,keeping Rule 4 at the forefront of your mind.

First add a letter to word 1 to form a five-letter word 2. Rule 4 forbids us from addingagain just yet, so we substitute to form word 3, leaving it with five letters also. Now wecan add again, so that word 4 consists of six letters. Substitute one more time, and theresult is a six-letter word 5. If you chose to substitute first and then add, you’d stillcome up with six as the maximum number of letters for word 5.

• Often, the work you put into earlier questions will help you answer later ones.For example, by the time you get to this question, the fact that you have toalternate performing the desired operation (adding a letter for a maximum, ordropping a letter for a minimum) should be clear from your work on question 16.

Page 35: 10-PrepTest 10 Explsdl.keywin.org/1/e/1e1103afee7eee195fe5fbe511587657.pdfNo part of this book may be reproduced in any form, by photostat, microfilm, xerography or any other means,

KAPLAN LSAT PREP _______________________________________________________ LSAT Test X Explained: Section II

© K A P L A N 33

GAME 4 — Guitar Concertos(Q. 19-24)

The Action: This one’s a hybrid game: a lot of grouping, with a touch of sequencing.We must choose two concertos from each of three composers (six total), and place themin the order of the six Sundays in which they’re played. The Key Issues to look for arepretty standard grouping and sequencing factors, such as:

1) Which concertos can be selected together?

2) Which concertos can’t be selected together?

3) On which Sundays can each selected concerto be played?

The Initial Setup: We can prepare by placing the six concert days on our page, alongwith the concertos listed by composer. Be sure to indicate the precise number of worksfrom each composer that must be chosen:

Pick 2 from each: GIU ROD VIVHJK MNOP XYZ

1 2 3 4 5 6

The Rules:

1) Pretty standard; If N, then J. Don’t make the mistake of inferring the opposite —selecting J does not require selecting N. But you can infer the contrapositive — if J isn’tchosen, then N can’t be either.

2) If M, then no J and no O. Some might stop here and combine the two J rules, but we’llkeep going and get back to them. Do note the contrapositive: If either J or O is chosen,then M can’t be.

3) Exactly parallel to Rule 2, except with different concertos: If X, then no Z, and no P.And an exactly parallel contrapositive, too: If either Z or P is chosen, X isn’t.

4) This rule introduces the first sequencing element to the game — if J and O are twoout of the six concertos chosen, then J must be played before O. You could shorthandthis as [IF J and O, then J. . .O].

5) A statement in the form “X cannot so&so unless R” is equivalent to “No X unless R,”and thus is equal to “If X then R.” So if you wanted to jot this down, [If X = 5 then M, N,

Page 36: 10-PrepTest 10 Explsdl.keywin.org/1/e/1e1103afee7eee195fe5fbe511587657.pdfNo part of this book may be reproduced in any form, by photostat, microfilm, xerography or any other means,

KAPLAN LSAT PREP _______________________________________________________ LSAT Test X Explained: Section II

34 © K A P L A N

O, P = 1] would suffice, those four letters representing the Rodrigo pieces. Rememberto think through (if not jot down) the contrapositive as well: If something other than M,N, O, or P is 1st, then X can’t be 5th. Of course, in terms of drawing, you might simplyhave chosen to circle this rule on your page to remind you it’s there. But the thinking —the thinking has to get done.

Key Deductions: Not much to be done. So many of the rules are hypothetical — that is,introduced with if clauses — that there’s very little that can be certainly determined.You might have noticed that the inclusion of J in both Rules 1 and 2 is a hint that thesetwo rules can be combined. N won’t be selected without J, while M won’t be selectedwith J; clearly a conflict of interest between M and N. So if you like, you can add N tothe list of concertos that can’t be selected with M. Beyond that, however, we’ll wait forthe questions to give us concrete information with which to apply the rules.

The Final Visualization: Here’s everything we now know about the game:

Pick 2 from each:

1 2 3 4 5 6

GIU ROD VIVHJK MNOP XYZ

Rules

If N ➔ J and if no J ➔ no N

If M ➔ no J, no O, no N

and if J or O or N ➔ no M

If X ➔ no Z and if Z ➔ no X

If J O ➔ J … O

If X 5th ➔ M or N or O or P = 1stThe Big Picture:

• It can’t be stressed often enough: Take the time at the beginning of the game tosearch for ways in which to combine the rules. In the previous game, the rulesdidn’t relate well, but in the other three on this section, making the KeyDeductions was crucial. Don’t worry about the minute or two you lose; you’llmore than make up for it at the back end.

• Don’t view every game as an entity unto itself — every sequencing/groupingpractice game you’ve done should bring something to bear in your effort to tacklethis one. On your test, turning the page and coming face to face with a game asfamiliar as this one should boost your confidence and help you to settle into agroove.

• Keep in mind at all times the dual nature of the game. In every question, askyourself whether the ordering of the entities or the selection of the entities is therelevant issue. This will help you to focus on the applicable rules anddeductions.

Page 37: 10-PrepTest 10 Explsdl.keywin.org/1/e/1e1103afee7eee195fe5fbe511587657.pdfNo part of this book may be reproduced in any form, by photostat, microfilm, xerography or any other means,

KAPLAN LSAT PREP _______________________________________________________ LSAT Test X Explained: Section II

© K A P L A N 35

• When the majority of the rules are if/then rules, don’t break your back trying todeduce all possibilities. Think through each one as well as its contrapositive; jotit down for easy and accurate reference; and wait for the questions to give youmore concrete data.

The Questions:

19. (B)(A) violates Rule 1, by including J without N. (C) violates Rule 2, by selecting J with M.(D) is O.K. on Rule 5, but violates Rule 3 (by selecting both X and P). And (E) goeswrong by violating Rule 4: To work, (E) would have to reverse J and O. We’re left withthe one answer that violates nothing, choice (B)>

• As always, in acceptability questions, you can check the rules against the choicesand quickly cross off the non-conformers.

20. (C)Obviously, since we’re told which six concertos are chosen, the grouping aspect isalready decided; we must therefore focus on the sequencing element, which meansRules 4 and 5. Rule 4 seems to apply, since both J and O are chosen. If, as given, N isplayed on the first Sunday, O couldn’t be second, because that would place O before J;a violation of Rule 4. It’s as simple as that. Nothing forbids the concertos in the otherfour choices from being played on the second Sunday.

• Use your instinct: If you can narrow down which rules are relevant to a particularquestion, based on the “action” suggested by the stem, and then let the specificsof the question lead you to the exact rule that applies, your work becomes mucheasier.

21. (E)Once again, the action suggests sequencing, and not surprisingly, J and O appear in thestem. Note also that the question asks which concerto can’t be fifth. Fifth, huh? Doesthat ring a bell? It should: Rule 5 explicitly states “X cannot be played on the fifthSunday....” So (E) is a good bet, but let’s make sure.

If J, O, and Y are the first three to be played, O can’t be first without violating Rule 4. Soeither J, a Giuliani concerto, or Y, a Vivaldi piece, must be played on the first Sunday.But for X to be fifth, according to Rule 5, a Rodrigo concerto must be played first, whichis impossible, so our suspicion is confirmed — X, choice (E), can’t be fifth under theseconditions.

• Use your instinct, Part II: Pick up on situations in the question stem that closelymimic the wording of the rules. If there’s a whole rule devoted to which entitycan’t be fifth under certain circumstances, and then a question comes alongasking “Which can’t be fifth?,” then that’s a good indication that that entity willbe the culprit.

Page 38: 10-PrepTest 10 Explsdl.keywin.org/1/e/1e1103afee7eee195fe5fbe511587657.pdfNo part of this book may be reproduced in any form, by photostat, microfilm, xerography or any other means,

KAPLAN LSAT PREP _______________________________________________________ LSAT Test X Explained: Section II

36 © K A P L A N

22. (B)Building on the J/O sequencing theme of the last two questions, O first means that Jmust not be chosen at all — so much for (A). Without J, the other two Giulianiconcertos, H and K, must be chosen, which means that either could be our answer. Ofcourse, only one of them can be listed in the choices, and it’s K, choice (B).

As for the others, Rule 2 won’t allow M and O to be chosen together, which kills (C).Without J, N must be left out also, according to Rule 1, eliminating (D). And without Mand N, P must be chosen along with O to represent the two Rodrigo concertos, whichmeans that X must be left out, thanks to Rule 3.

• Do only as much work as necessary to answer the question. As soon as youdeduce “no J, which means H and K must be selected,” scan the choices to see ifeither one is there. Since Logic Games answers are objectively correct, and time issuch a critical factor on this section, have the confidence to stop right there, circle(B), and move on.

23. (D)Check your rules; J and O must be left out if M is selected, and Z and P must be left outif X is selected. This allows us to immediately cross off J, O, and Z, choices (A), (C) and(E), respectively. That leaves K and Y. If X is selected, Z can’t be (Rule 3), and thereforeY must be chosen. If X isn’t selected, then the other two Vivaldis, Y and Z, must beplayed. In either case, Y must be chosen. Meanwhile, here’s an acceptable arrangementwhere K, choice (B) could be left out: H, J, N, O, Y, Z.

• When asked which must be selected in the absence of an if-clause (unless you’vespecifically deduced this beforehand), often the fastest approach is to throw outthe ones that you know could be left out.

• Sometimes questions that don’t offer hypothetical information are tough, but ifyou know where to look, you can often at least narrow the choices down to threeor even two. Even if you can’t go any further, or are running out of time, the oddsare still significantly improved if you have to guess.

24. (A)This one’s straight out of the Key Deduction we came up with earlier by combiningRules 1 and 2: N demands J, whereas M can’t be chosen with J. Selecting M and Ntogether is therefore impossible. The concertos in all of the other choices work finealongside N.

• Don’t be surprised when you encounter easy questions at the end of a game or asection; they’re scattered around the section just like the difficult ones.

Page 39: 10-PrepTest 10 Explsdl.keywin.org/1/e/1e1103afee7eee195fe5fbe511587657.pdfNo part of this book may be reproduced in any form, by photostat, microfilm, xerography or any other means,

© K A P L A N 37

SECTION III:READING COMPREHENSION

Page 40: 10-PrepTest 10 Explsdl.keywin.org/1/e/1e1103afee7eee195fe5fbe511587657.pdfNo part of this book may be reproduced in any form, by photostat, microfilm, xerography or any other means,

KAPLAN LSAT PREP ______________________________________________________ LSAT Test X Explained: Section III

38 © K A P L A N

PASSAGE 1 — New Crude Pumps(Q. 1-8)

Topic and Scope: Crude oil pumping technology; specifically, new technologies thatmay make offshore processing platforms obsolete.

Purpose and Main Idea: Author wants to explain the difficulties inherent in thecurrent system of processing oil offshore, and to sketch two possible technical solutionsfor those difficulties. This passage doesn’t have a traditional “main idea”—that is, apersonal thesis that the author has developed and is trying to prove—though that didn’tstop the testmakers from asking Question 1, now did it?

Paragraph Structure: ¶ 1 outlines the problem and the solution, both of which hingeon what oil pumps can and cannot do. It’s easy to get lost in all of the technical minutiaeabout crude oil; the most important thing is lines 16-19, the clearest statement of thecurrent dilemma. ¶ 2: Definitions, pros, and cons of the two pumps under currentredesign (here we’ll call them the “p-d pump” and the “c-pump”). ¶ 3: A coda brieflysketching out the kind of further work that’s being done.

The Big Picture:

• A lot of students saved this passage till later in the Reading Comp. section,doubtless because of the unfamiliarity of the subject matter. That the questionsturned out to be relatively easy doesn’t make the decision to postpone an unwiseone. Go with your instincts! Better that, than hitting the passages in order justbecause that’s how they’re printed there.

• Many passages begin as this one does, announcing a Big Problem and exploringone or more possible solutions. When that happens, keep the scope of theproblem in mind (wrong choices are likely to go off into other, related areas) andmake sure you notice which solution(s), if any, the author favors.

• When passages are technically based, as this one is, make a mental picture of thesituation. Even if your mental picture of the offshore platform, the crude comingup, and the pumps and pipelines is wildly at odds with real life, who cares? It’sgoing to make the unfamiliar technical matters a lot more comprehensible.

Page 41: 10-PrepTest 10 Explsdl.keywin.org/1/e/1e1103afee7eee195fe5fbe511587657.pdfNo part of this book may be reproduced in any form, by photostat, microfilm, xerography or any other means,

KAPLAN LSAT PREP ______________________________________________________ LSAT Test X Explained: Section III

© K A P L A N 39

The Questions:

1. (A)(A) reflects the problem announced early on, the experimental nature of the redesign ofthe two pumps, and the potentiality (“may help diminish the danger toworkers. . . ”) of the whole business.

(B) distorts the final sentence, which suggests that underwater pumps may lead to thedesirable result of onshore processing. The oil companies want to get the processing outof the ocean and onto dry land. A mental picture of the situation has to rule this choiceout.

(C) The infamous half-right half-wrong choice: The first eight words are fine, but it’s thedesire to reduce danger and maybe eliminate platforms, not to enhance efficiency, thatis driving the research described. Also, (C) fails to mention oil!

(D) also fails to mention oil, and misses the fact that it’s offshore processing they want tomake obsolete.

(E) first of all blows up the gas/liquid details into the main idea. Also, the authorsuggests that the separation of gas and oil either happens naturally (lines 23-25) orthrough technology (lines 44-47), so the quest described in (E) seems unnecessary.Finally, of course, separating liquid and gas isn’t the reason why oil processing isdangerous in the first place.

• The answer to “global” questions has to cover the same topic and scope as theoverall passage itself. Read choices completely. Beware of answers to “global”questions that fixate on details.

2. (D)This choice is easily reached if you ignore all the various passage references to“pressure” and just look for what the question’s about: natural pressure. That’s lines 13-16.

(A) seems to play off the reference to reductions in pressure in lines 46-47. But that isn’tnatural pressure but rather head pressure, pressure within a pump (that was only builtin the first place because natural pressure is inadequate to the task).

(B) The passage refers to the constancy of oil pumps currently in use (lines 16-19), notto the constancy or inconstancy of natural pressure. In any case, based on the volatilenature of crude oil as described in lines 19-23, natural pressure is inferably far from aconstant force.

(C) Give credit where credit is due. Natural pressure may not be much, but it is strongenough to send the oil past the wellhead to the platform, located above (lines 14-16).

(E) The ability of natural pressure to send oil to shore would solve everyone’sproblems. But that’s why they need pumps: because nature isn’t sufficient.

Page 42: 10-PrepTest 10 Explsdl.keywin.org/1/e/1e1103afee7eee195fe5fbe511587657.pdfNo part of this book may be reproduced in any form, by photostat, microfilm, xerography or any other means,

KAPLAN LSAT PREP ______________________________________________________ LSAT Test X Explained: Section III

40 © K A P L A N

• Read each question stem carefully, and milk it for all possible help. Moststudents, desperately out of control, only get a glimpse of each question, which iswhy their Reading Comp. scores and timing are so rotten.

3. (B)(B) is saying, in so many words, what we described under “Paragraph Structure,”above. The two new pumps, alluded to in line 10, are described and evaluated in ¶ 2 aspossible solutions to the problems outlined in ¶ 1.

(A) Half-right, half-wrong. Concrete detail, yes, but ¶ 1 presents no real argument,flawed or otherwise—just a description of a problem and its details.

(C) Also half-right, up through the phrase “pros and cons.” It’s the pros and cons of thetwo new pumps that are described, not a ¶ 1 claim.

(D) “Detracts” is way off—¶ 2 advances the passage, rather—and it’s just not accurate towrite off ¶ 2 as a presentation of “problems.” It’s ¶ 1 that lists problems. ¶ 2 assesses twopossible ways of solving ‘em.

(E) ¶ 2 “modifies” nothing, and details no con viewpoints.

• Be wary when, as here, all five choices present you with abstract text. Read very,very carefully, and lean heavily on your pre-phrasing of the answer.

4. (E)Choice (E) couldn’t be more clearly conveyed than by line 26!

(A) This causes loss of “head”—lines 25-26.

(B), (C) Neither is given a name in the passage.

(D) The impeller is exclusive to the c-pump. This choice is some sort of distortion oflines 45-47.

• Don’t be suspicious (I was going to say, don’t lose your head) if the answer seems“too easy.” The testmakers do hand you some points on a platter.

5. (C)This is a pretty straightforward paraphrase of lines 5-12.

(A) In the context of lines 8-12, the word “even” (line 10) means that however desirableit is to reduce the need for human labor on platforms, eliminating platforms is betterstill. (A) contradicts this.

Page 43: 10-PrepTest 10 Explsdl.keywin.org/1/e/1e1103afee7eee195fe5fbe511587657.pdfNo part of this book may be reproduced in any form, by photostat, microfilm, xerography or any other means,

KAPLAN LSAT PREP ______________________________________________________ LSAT Test X Explained: Section III

© K A P L A N 41

(B) Getting people off platforms is desirable because explosions and hurricanes andsuch make platform work dangerous, not because we don’t know enough abouttransporting crude.

(D) implies that the author gives thumbs up to the p-d pump, but in fact the passageconsiders both pumps about equal in terms of pros and cons. And according to ¶ 3,platforms will become superfluous when either pump is successfully placedunderwater.

(E) contradicts the sense clearly conveyed in ¶ 3 that getting rid of offshore platformshas been, and remains, a desirable goal.

• When (as here) there’s no alternative to looking over all five choices, be ruthlessin rejecting choices that run counter to the topic, scope, and overall authorialpoint of view.

6. (B)Details about current pumps have to come out of ¶ 1. If you reread or just skim lines16-29, you get the picture that (B) paints: The sudden changes in phase states, fromliquid to gas to solid and back again, reduce pump pressure and thus make itimpossible for the pumps to get the crude to shore. Inferably, if those changes could becontrolled, pump efficiency would be improved (as the p-d pump, in particular, hashad success doing).

(A) Gas is never listed as a substance that can, let alone should, be removed.

(C) Au contraire. Such changes decrease efficiency.

(D) Au contraire. It does too.

(E) sounds plausible—and scary; but the only passage reference to explosions is at line 7and no explanation for same is every provided.

• The harder you have to work to justify a choice, the more likely it is to beincorrect.

• Let the passage structure tell you where to seek out a particular answer. Since ¶ 2covers the pumps of the future, it’s got to be the previous ¶ that provides theanswer to Q.6.

7. (E)Appears to be a detail question, but has as much to do with inference and tone asknowing the specifics about each pump. (E) describes the downside of the p-d pumpand, inferably, the reason that the c-pump is seen as “the alternative” with the “provendesign” (see lines 33-41).

Page 44: 10-PrepTest 10 Explsdl.keywin.org/1/e/1e1103afee7eee195fe5fbe511587657.pdfNo part of this book may be reproduced in any form, by photostat, microfilm, xerography or any other means,

KAPLAN LSAT PREP ______________________________________________________ LSAT Test X Explained: Section III

42 © K A P L A N

(A) is right in noting that the c-pump is cheaper and lower maintenance, but wrong inciting the p-d pump as “more promising.” The author makes no such assessment. Eachpump described has its pros and cons.

(B), too, errs by implying that the author favors the p-d pump over the other one. Also, it’sthe c-pump, not the p-d pump, that “has been used in other settings” (lines 41-43).

(C) Boy, did students pick this one. Two problems, either of which is enough to disqualifyit: (1) The p-d pump does in fact have a single or twin screw, but there’s no evidence thatsuch a device is unique to that pump; maybe the c-pump has something like it as well. Sothis isn’t a clear-cut point of difference. (2) The only device mentioned in the passage as“sucking fluid in at one end” is the c-pump. We can’t be sure that that’s what the screws doin the p-d pump. (In fact, from the sound of it, the p-d pump does nothing of the kind.)Again—not a clear-cut point of difference.

(D) The liquid/gas shifts, which are of course discussed a paragraph earlier, are commonto all crude and affect both pumps under redesign.

• Read every choice carefully, especially in inference questions. Here, many chose (C)because they recognized the detail about the screws and never really looked closelyat (E).

8. (D)(D) echoes the sentiments of the very first sentence. The dangerous platforms can’t be doneaway with yet, because the technology still doesn’t exist to pump the crude on shore andprocess it there; that’s the problem driving all of the research that the author describes.

(A) Crude “may” (not must) “consist of oil and natural gas in multiphase states.” And thepassage never suggests that technology causes crude to be transported to shore that way.

(B) The volatility of the shifts in crude content makes it highly unlikely that a 50-50 gas-liquid split could be maintained. In any event, crude doesn’t “reach the wellhead inconstant proportions” (lines 22-23), and there’s no sense that altering those proportions is anecessity.

(C) The problems of the p-d pump suggest that large proportions of corrosive material areinevitable—that’s why they’re fiddling around with the design of the pumps.

(E) Seems plausible as you picture liquid crude moving through the pipes, but as far asthe passage informs us, processed crude can move to shore in any combination of solid,liquid, and gas. (We’re never really told what “processing” consists of, but it need notmean turning lumpy crude to pure liquid.)

• Quite often, a question that comes at the end of the group will hinge on somethingmentioned early in the passage. The testmakers want to check whether you’re awakeor forgetful. So the message should be clear: Always be aware of each piece of thepuzzle, and how the author is fitting them together. That awareness will enable youto check anywhere in the passage for the information needed for each question.

Page 45: 10-PrepTest 10 Explsdl.keywin.org/1/e/1e1103afee7eee195fe5fbe511587657.pdfNo part of this book may be reproduced in any form, by photostat, microfilm, xerography or any other means,

KAPLAN LSAT PREP ______________________________________________________ LSAT Test X Explained: Section III

© K A P L A N 43

PASSAGE 2 — Venetian Religious Narrative Painting(Q. 9-14)

Topic and Scope: 15th century Italian narrative painting; specifically, the differencesbetween the Venetian version (as recently analyzed by a critic) and the Tuscan.

Purpose and Main Idea: Author wants to explain why the Venetian approach toreligious narrative painting in the 15th century is so different from the Tuscanapproach, and lists a reason cited by critic Brown (differences in attitude) and othersthat Brown doesn’t cite (differences in climate and artistic skill).

Paragraph Structure: ¶ 1 lists characteristics of the two types of narrative religiouspainting, and reports Brown’s explanation for the difference: Unlike the Tuscans’ large-scale, simple style, the Venetian style added meticulous detail and depicted actualcontemporaries because doing so made the art more authentic and distinctive.

¶ 2 focuses on the climate issue that Brown ignores. Venetian dampness discouragedthe kind of large scale frescoes in which the Tuscans specialized, so the Venetians reliedon a different, smaller scale model. ¶ 3 points out a third influence: The skillfulpainting of people was prized in Tuscany but rare in Venice, which instead valued theskillful painting of buildings. Hence we get Tuscan frescoes where the human subjectsare large and well-executed, and Venetian paintings that downplay the human elementand emphasize the physical environment.

The Big Picture:

• When a passage is strongly based on contrast, you want to be clear about thedifferences between the entities in question, but don’t endeavor to master them indetail. Get a broad sense of the contrast.

• Sometimes making a quick list off to the side can help. Tuscany: Frescoes, largescale, simple, focus on people. Venice: No frescoes, small scale, embroidered,focus on buildings, depicted real people.

The Questions:

9. (B)This one has the right focus on explanation (which is after all the author’s purpose), anddeftly sums up the ¶ structure by alluding to the historical (¶ 1) and other (¶ 2 & 3)factors. Some students were bothered by the failure of (B) to mention Tuscany, althoughthey might have realized that from the very first sentence, the author uses Tuscanreferences mostly for background, to put the Venetian details in sharper relief. Notethat P.F. Brown’s book, the starting point for the passage, is about Venice, not Tuscany;note also that Venice or Venetians are mentioned 15 times, Tuscany only five, with only16 lines out of 64 (25%) devoted to information about Tuscany. Of course, if botheredby (B), you also have to factor in the inadequacies of the wrong choices, especially thefailure of all four to get the scope right:

Page 46: 10-PrepTest 10 Explsdl.keywin.org/1/e/1e1103afee7eee195fe5fbe511587657.pdfNo part of this book may be reproduced in any form, by photostat, microfilm, xerography or any other means,

KAPLAN LSAT PREP ______________________________________________________ LSAT Test X Explained: Section III

44 © K A P L A N

(A) ignores Venice altogether—and if you’re going to omit a painting style from themain idea of this passage, it had better not be Venice. Also, (A) focuses only on thedetails about painting people into frescoes, lines 31-37, far too limited in scope for amain idea answer.

(C), too, is too narrow in reducing the Venice/Tuscany differences to a matter of“authentic detail”: The author takes pains to cite many other differences, includingscale and local artistic preferences. Also, the author restricts the scope to Venetianreligious narrative paintings, a point respected by (B) but lost on (C).

(D)’s scope is even worse, failing to place the passage in the 15th century and focusingon ¶ 3 only. If all that weren’t bad enough, (D) even distorts the point made in ¶ 3,which is that Tuscany and Venice prized different aspects of art, not that Venetianpainters were more skilled than the Tuscan ones.

(E) is pretty much just a paraphrase of lines 42-48, except for distorting that detail bycalling the palace cycle the “primary” influence on the paintings.

• You’re asked to pick the best of five choices, not the best of all possible ones.Don’t argue with choices: Compare them.

• Don’t let your misgivings about a choice blind you to the potentially much biggerproblems with other choices. Letting your quibbles with the credited answer leadyou to choose a manifestly worse answer is something you should constantly beon guard against.

10. (C)(C) more or less translates Q. 9(B) into abstract language (note the use of the noun“factors” and verb “explain” in both), albeit explicitly alluding to the “difference inpainting styles” that Q.9(B) sidestepped.

(A) inaccurately accuses the author of playing favorites. Not a scintilla of evidenceindicates a preference for Tuscany or Venice. To compare two things is not necessarilyto show a preference for one over the other.

(B) What “conventionally held view”? The critics’ view of Tuscan painting as cited inthe first sentence?—but the author never challenges that view. Some earlierinterpretation of the Venetian style?—but no such interpretation is mentioned. P.F.Brown’s?—but that view is cited approvingly, not targeted for “reevaluation.”

(D) is poor on three counts: (1) The author is describing styles, not evaluating them, andnever discusses strengths vs. weaknesses. (2) No “opposing views” are presented: Theauthor doesn’t disagree with Brown’s explanation of the Venetian style in ¶ 1, butsimply enlarges upon it. (3) “Evolution” implies change over time, something theauthor never gets into.

Page 47: 10-PrepTest 10 Explsdl.keywin.org/1/e/1e1103afee7eee195fe5fbe511587657.pdfNo part of this book may be reproduced in any form, by photostat, microfilm, xerography or any other means,

KAPLAN LSAT PREP ______________________________________________________ LSAT Test X Explained: Section III

© K A P L A N 45

(E), again, misrepresents the passage as a clash of theories or viewpoints. It is not. Twodifferent painting styles are characterized and explained. Period.

• When two questions (like 9 and 10 here) seem similar, consider working on themtogether. And perhaps the later question may be easier than the earlier one.Remember you always have the option to attack questions out of sequence.Exercise that option at will!

11. (B)Turns out to be a variation on Logical Reasoning “assumption” questions. Brown’sexplanation is ¶ 1 only, and the eyewitness style appears in lines 20-26. Her allegationthat the eyewitness style was influenced by a contemporary writing style assumes, ofcourse, that writing can influence painting. Which is all that (B) is saying.

(A) refers to architecture and drawing skill, which are brought up only in ¶ 3, by whichpoint the eyewitness style—and Brown herself—have long since been left behind.

(C) refers to the Tuscan frescoes, which only come up in ¶ 2 and are an issue that Brownneglects. Moreover (C) contradicts the whole sense of the passage that the Venetians hadinfluences separate from the Tuscans, which is why the two painting styles turned out sodissimilar.

(D) Nice try, but the detail about the palace art comes from the wrong ¶, ¶ 2. At thatpoint we are no longer discussing Brown’s views but rather the factors that Brown leftout. (D) does sound like something the author would agree with, but cannot be read asinferable from Brown’s argument.

(E), like (C), implies cross-fertilization of art from region to region, contrary to thepassage’s sense that art in Tuscany and Venice developed under separate influences.Also, of course, no such “influx” is referred to anywhere near lines 20-26.

• Use whatever clues the stems provide in order to localize your search of thepassage. For instance, skimming ¶ 1 for the phrase “eyewitness style” allows youto locate the particular chunk of passage that will yield the right answer.

• Keep your Logical Reasoning skills handy during Reading Comp., too. Questionsabout the logic of a passage will benefit from them, especially the tactic ofidentifying author assumptions.

Page 48: 10-PrepTest 10 Explsdl.keywin.org/1/e/1e1103afee7eee195fe5fbe511587657.pdfNo part of this book may be reproduced in any form, by photostat, microfilm, xerography or any other means,

KAPLAN LSAT PREP ______________________________________________________ LSAT Test X Explained: Section III

46 © K A P L A N

12. (E)The correct answer could come from any part of the passage (since Venetian artists arementioned in every ¶), and here (E) comes from ¶ 3. We’re explicitly told that in Venice,the ability to draw people well “was acquired and appreciated much later” (lines 56-57)than the 15th century, while “painting architecture in perspective was seen as aparticular test of the Venetian painter’s skill” (lines 63-64). The contrast is summed upby (E).

(A) What Venetian apprentices in Tuscany? Never mentioned. Such apprentices, if theyexisted, might explain how Venetians acquired drawing skill “much later” than the 15thcentury—but now we’ve really wandered beyond the scope of passage and question.

(B) No, this is a Tuscany characteristic (lines 31-36). Venetian paintings had a lot of tinydetails conducive to close viewing, remember?

(C) ??? For all we know, the artists in the stem and (C) were one and the same. The onlypassage reference to “influences” is, as we saw in Q. 11(B), the influence of historicalwriters on painters.

(D) No, the reason they didn’t paint frescoes was that the Venetian climate precludedsame.

• If your reading of the question stem doesn’t immediately tell you where to searchfor the answer, consider saving that question until later on—it’s almost certain totake up more time.

13. (A)The question’s focus on buildings has to direct you to ¶ 3, whose last sentence is almostverbatim the question stem and (A).

(B) There’s nothing inherent in religious stories, as far as we know, that would leadpainters to include elaborate buildings in their work. Besides which, it’s the Tuscanpaintings that were the more religious; Venetian paintings were more secular in style(lines 31-34 and 42-45).

(C) is clever, in that it makes sense that buildings would be ideal subjects in a large-scale form like the fresco. But frescoes were Tuscan, not Venetian — or had youforgotten?

(D) No comparison between Venetian and Tuscan architecture is made or alluded to.For all we know, both types of building were elaborate.

• Halfway through the questions on any passage, take a few seconds to remindyourself of the paragraph structure and recap some of the key ideas. Here, forinstance, if you don’t stop to look over the list we compiled of Venetian andTuscan art traits, you may get confused and fall for wrong choices like 12(B) or13(C).

Page 49: 10-PrepTest 10 Explsdl.keywin.org/1/e/1e1103afee7eee195fe5fbe511587657.pdfNo part of this book may be reproduced in any form, by photostat, microfilm, xerography or any other means,

KAPLAN LSAT PREP ______________________________________________________ LSAT Test X Explained: Section III

© K A P L A N 47

14. (C)This one explicitly rebuts the author’s assumption that the Venetian artists had little orno exposure to frescoes and the painting style associated with frescoes. (C) woulddeepen the “mystery” that the author wants to solve, namely: how come the two cities’styles were so different?

(A) is specifically stated by the author in lines 42-48, so there’s no way that it couldweaken any part of the author’s argument.

(B) That Venetian historical writing might be similar in one major respect (the inclusionof secular detail) to Tuscan secular painting has no impact on the author’s claim aboutVenetians’ ignorance of large-scale religious paintings.

(D) is eminently consistent with the passage, at least in its allusion to the strongreligiosity of Tuscan paintings; in any event, (D) makes no reference to Venice orVenetians and so could not weaken the claim in question.

(E) The passage states that skill in drawing the human figure came to Venice “muchlater” than the 15th century. If, as (E) states, “much later” means the late 1600’s, so what?That’s eminently consistent with the text and has no effect on the contention in question.

• Weakening an argument in Reading Comp. is little different from doing so inLogical Reasoning (it may be a bit easier here in Reading Comp.). Consider whatthe author assumes, and check whether an answer choice contradicts a key pieceof evidence. Also, look for wrong choices that either strengthen the argument orgo outside the scope.

Page 50: 10-PrepTest 10 Explsdl.keywin.org/1/e/1e1103afee7eee195fe5fbe511587657.pdfNo part of this book may be reproduced in any form, by photostat, microfilm, xerography or any other means,

KAPLAN LSAT PREP ______________________________________________________ LSAT Test X Explained: Section III

48 © K A P L A N

PASSAGE 3 — Legal Realism(Q. 15-21)

Topic and Scope: The vagueness of the law; specifically, the view of a group of legalphilosophers called “realists” that the law is inherently indeterminate.

Purpose and Main Idea: Instead of providing his own argument, the author’s purposeis to communicate the argument that the realists made, the main idea that’s summed upin line 11: “Indeterminacy pervades every part of the law.”

Paragraph Structure: ¶ 1 begins with the current view about which “legal scholarsagree” (line 1): There’s some vagueness in the way law is written and hence applied, butthe foundation for the system is basically solid and based on clarity. The “earlier” (nohint about how much earlier) realists begged to differ.

¶ 2 uses the example about the aunt and niece to illustrate the number of different rulesthat might apply in a given case, and thus the realists’ first point—that any vagueness inany of those rules will contribute to the indeterminacy of the outcome.

¶ 3’s key signal is at line 22: “an even more damaging kind of indeterminacy” tells us (ifthe sheer length of ¶ 3 didn’t already do so) that this is the meat of the realists’argument, taking us beyond the vagueness of language to the uncertainty of precedent.First we get the factual background: Judges’ decisions, meant to be binding precedents,are composed of holding and dicta. The holding is what later judges are supposed to beguided by, while the dicta are the inessential points of law. Yet according to the realists,this holding/dicta distinction is observed only loosely. Judges are free to redefine eachin their subsequent opinions, and in the end a single court case can create multiple andcontradictory precedents—hence, indeterminacy for judges who are presumablylooking for concrete precedents to rely upon.

The Big Picture:

• While most passages consist of the author’s argument, be ready for passagesduring which the author offers no personal point of view and instead reports onsomeone else’s argument.

• The lengthy ¶ 3 offers a fairly typical LSAT ploy. Between lines 26-46 there’s a lotof complex detail, and it’s easy to get bogged down. Yet if you just hang in thereuntil line 46, the word “then” comes along to signal a conclusion, and the last twosentences sum it all up pretty clearly. The point? When in doubt, keep reading!The author will often restate matters more simply and clearly later on.

Page 51: 10-PrepTest 10 Explsdl.keywin.org/1/e/1e1103afee7eee195fe5fbe511587657.pdfNo part of this book may be reproduced in any form, by photostat, microfilm, xerography or any other means,

KAPLAN LSAT PREP ______________________________________________________ LSAT Test X Explained: Section III

© K A P L A N 49

The Questions:

15. (E)The phrase “common-law system” appears first at line 24, but its second appearance(line 35) is the one that leads to the right answer. (E) is practically a verbatimrestatement of lines 34-36.

(A) That the common-law system allows judges to redefine legal issues and pick andchoose among them—which is what (E) implies—doesn’t mean that judges are findingearlier colleagues “in error.” In any event the specific issue of judicial error, whileperhaps familiar to you from everyday reading, never comes up here.

(B) Remember, you’re asked for something that “the realists argued.” They wouldnever claim that the legal system is “clear” in either theory or practice, and certainlydon’t mention judges’ “unawareness” as part of the problem.

(C) seems to distort lines 22-26. To the realists, the reliance of a common-law system onprecedent is the source not of “strength,” but of “an even more damaging kind ofindeterminacy.” Moreover (C)’s distinction between precedent and abstract principlesis bogus. Insofar as we can tell from the passage, a legal precedent, while based on aspecific court case, would in fact become an “abstract” principle of law on which otherjudges should rely.

(D) The realists made no charge of “willful misinterpretation.” To them, the leeway thatjudges have to redefine precedents is built into the system, not the result of judges’misconduct. Moreover, the author never reports on how the realists would “improve”the system (if indeed that was ever part of their scope; they were philosophers, after all,not necessarily legal reformers).

• When a question cites a specific phrase from the passage like “common-lawsystem” here—or “eyewitness style” in Q.11—use it like a line reference, to helpyou find the answer. And don’t jump at the first such reference you see! Bethorough.

16. (E)This question clearly sends you to the first half of ¶ 3, where we learn that the holding isone of the two parts of the decision (the dicta are the other part). (E)’s verbs (“sets forth”and “justifies”) act neatly to paraphrase lines 28-30.

(A) is right in alleging that the decision is supposed to be binding on later judges (lines24-26), but wrong in contrasting that with the holding. The holding is supposed to bethe binding part (lines 30-31).

(B) and (C) are both flat-out incorrect. The holding both announces and explains thedecision.

(D) Close but no cigar. The decision consists of the holding and the dicta.

Page 52: 10-PrepTest 10 Explsdl.keywin.org/1/e/1e1103afee7eee195fe5fbe511587657.pdfNo part of this book may be reproduced in any form, by photostat, microfilm, xerography or any other means,

KAPLAN LSAT PREP ______________________________________________________ LSAT Test X Explained: Section III

50 © K A P L A N

• A true “detail question,” like this one, demands care. Don’t let time constraintsforce you to read sloppily and incompletely.

17. (B)As baffling as the question stem looks and reads, the answer is pretty straightforward.Indeed, (B) in a sense sums up the realists’ “main idea” about the effect of precedent onjudges—it can lead to vast differences of opinion. The relevant portion of the passage islines 46-54.

(A) The first half is o.k.—although (B)’s “might differ” is better than (A)’s morecategorical “would most likely disagree”—but the “overturning” of decisions is nevermentioned.

(C) contradicts the last sentence of the passage. Far from feeling bound by every rule(which sounds like a recipe for paralysis), the judge may pick and choose among therules. That’s the “indeterminacy” that the realists are getting at.

(D) Hardly. That “lack of unanimity” leads to the judge’s ability to choose among therelevant rules, not to the absence of precedent.

• By all means, when a stem is as forbidding as this one, save it till later in thesection. You’ll find that you’re more relaxed and can concentrate better on thequestion, when a lot of the other questions are out of the way.

18. (A)Today’s legal scholars only enter the passage up front, and the very first sentencedemonstrates the point of agreement between them and the realists: Indeterminacy, atleast the indeterminacy stemming from linguistic vagueness, is built into the law.

(B) is a realist notion. Lines 6-8 specify today’s scholars belief that most cases rest on“clear...meanings” and lead to “definite outcomes.”

(C) comes out of ¶ 3, where no mention of today’s scholars appears. We don’t knowwhether they hold the same view of the blurring of holding and dicta that the realistsdid.

(D) If “the boundaries of applicability of terms” means how rules are applied, then (D)contradicts ¶ 1: To today’s scholars, most cases end with “definite outcomes.” In anycase, (D) draws a distinction that appears nowhere in ¶ 1.

(E) Another realist notion that contradicts lines 6-8.

• We’ve said it before and we’ll say it again: The further you wander from the partof the passage where the answer has to be found, the more likely you are to runinto trouble. Here, if you move away from lines 1-8, you’re sunk.

Page 53: 10-PrepTest 10 Explsdl.keywin.org/1/e/1e1103afee7eee195fe5fbe511587657.pdfNo part of this book may be reproduced in any form, by photostat, microfilm, xerography or any other means,

KAPLAN LSAT PREP ______________________________________________________ LSAT Test X Explained: Section III

© K A P L A N 51

19. (D)This question gets at what the author means by the loose distinction between holdingand dicta. The latter are supposed to be the non-essentials in a case, that which did notcontribute to that case’s outcome. But if judges “are not bound by the original judge’sperception” (lines 40-41), and can “choose which rules of law formed the basis of thedecision in a precedential case” (lines 45-46), then inferably they can, as (D) suggests,pull from the dicta in deciding how the rules are to be applied.

(A) sounds plausible, since the passage mentions the holding and dicta as “parts” of thedecision. But note lines 37-38: The statement “even when the judge writing an opinioncharacterizes part of it as ‘the holding’” implies that making such a strict distinction isthe exception rather than the norm. In this light, also see wrong choice (E).

(B) is one more wrong choice harping on the rightness or “appropriateness” of judges’decisions. What judges disagree about, insofar as the realists are concerned, is what isand is not essential in a precedential case, not whether reasoning was or was notappropriate.

(C) The notion of a fixed consensus developing at any time is totally antithetical to therealists’ position.

(E) If this were true, then perhaps the blurring of holding and dicta would occur lessoften. The sense of the realists’ argument is, rather, that holding and dicta are easy toblur (“indeterminacy,” remember?).

• Keeping in mind the overall “spirit of the passage” can help you even onquestions that don’t appear to be global in nature. Here, remembering the generalpoint of view of the realists about judicial indeterminacy leads you towards (D),or at least allows you quickly to reject (A), (C), and (E).

20. (D)¶ 1 does indeed describe two viewpoints (the current view, and that of the earlierrealists) in brief, and then goes on to describe the latter at length.

(A) Neither the current view nor that of the realists can fairly be called “traditional.”(A) may be referring to the “traditional view” of holding and dicta, and the way thatthat fails to work out in practice. That seems a stretch; but even if you accept thatreading of (A), it only describes ¶ 3, not the passage structure overall.

(B) is easily rejectable in its assertion that two thought systems are compared “point bypoint.” From line 9 to the end, we are concerned with the realists’ argument only.

(C) If the concept that (C) is allegedly defining is judicial indeterminacy, then clearlythe rest of the passage is devoted to exploring it, not “refuting” it. In any case (C) missesthe idea of a point of view being described at length.

Page 54: 10-PrepTest 10 Explsdl.keywin.org/1/e/1e1103afee7eee195fe5fbe511587657.pdfNo part of this book may be reproduced in any form, by photostat, microfilm, xerography or any other means,

KAPLAN LSAT PREP ______________________________________________________ LSAT Test X Explained: Section III

52 © K A P L A N

(E) makes up a contrast between theory and practice that (as mentioned in terms of (A)above) may marginally relate to ¶ 3 but is inadequate as a summary of the overallstructure.

• To avoid getting all bollixed up when the answer choices are written in abstractterms, take more time to pre-phrase some sort of an answer before studying thechoices. Doing so will make you less vulnerable to the—shall we say?—indeterminacy of how such choices are written.

21. (D)In only a few words this choice manages to mention the topic (legal indeterminacy), thescope (the realists), and the purpose (view). Neat!

(A) Au contraire, the debate seems to have ended with the current agreement on thepart of legal scholars that is the subject of ¶ 1. The realists are clearly described as anearlier group, though we know not how much earlier.

(B) zeroes in way too much on ¶ 3—holding and dicta appear only at line 28—though“a distinction without a difference” is a glib way to summarize the rest of theparagraph.

(C) fails to notice that the scope of the passage broadens from line 9 onward toencompass other types of legal indeterminacy than the merely linguistic.

(E), like (B), focuses only on details of ¶ 3. Both choices sidestep the topic and scope.

• “Best Title” questions are rare on the LSAT. They’re little different from otherglobal questions in that they demand that you focus on the overall topic, scope,and purpose.

• Expect wrong choices in “Best Title” questions to focus on details, and beespecially wary of the clever title, the one with the most “punch”—it usually willnot be the one that does the best job of summarizing the whole.

Page 55: 10-PrepTest 10 Explsdl.keywin.org/1/e/1e1103afee7eee195fe5fbe511587657.pdfNo part of this book may be reproduced in any form, by photostat, microfilm, xerography or any other means,

KAPLAN LSAT PREP ______________________________________________________ LSAT Test X Explained: Section III

© K A P L A N 53

PASSAGE 4 — Civil Rights Movement Theories(Q. 22-27)

Topic and Scope: The theoretical underpinnings of the black civil rights movement;specifically, whether the classical social psychological model satisfactorily explains themovement and its success.

Purpose and Main Idea: Author wants to explain and evaluate the classical theory as itapplies to the civil rights movement, and ends up echoing the complaints of politicaltheorists that the classical theory doesn’t hit the mark.

Paragraph Structure: ¶ 1 announces the problem—the need to come up with atheoretical basis for the black civil rights movement—and identifies the two groupswhose views will be mentioned in the course of the passage (without, it should benoted, the author’s taking a position herself. Yet.)

¶ 2 focuses on the “classical” theory of social movements as expounded by socialpsychologists, and names three strains or formulations of that theory, although they goundefined for the moment. ¶ 2 ends with the conclusion to which all three lead: namely,that the civil rights movement was triggered by socioeconomic changes. We have totake that on faith until ¶ 3 tells us a little more about each of the three classical theoriesand the conclusion each leads to. As promised at the end of ¶ 2, they all point tosocioeconomic causes.

No sooner does ¶ 4 begin than the explanations in the previous two ¶’s are dismissed, atleast by political theorists, who believe (not surprisingly) that social psychologicaltheories blur the political factors underlying the movement, and that such theories makepredicting future social movements impossible. Finally in ¶ 5 the author herself takes astand, calling the political theorists’ objections “legitimate” (line 51) and evaluatingvarious tests that have been run on the classical theories. The results are inconclusive,but do not bode well for wider acceptance of the classical social psychological modelon the topic of the black civil rights movement.

The Big Picture:

• Unlike the previous passage, in which there were two points of view and theauthor stayed out of the debate, here we have three viewpoints and the authorstands up to be counted at the very end. These variations carry with them onestrong recommendation for your future LSAT work: Be ready for anything! Justmake sure you’re keeping an eye out for where the author stands.

Page 56: 10-PrepTest 10 Explsdl.keywin.org/1/e/1e1103afee7eee195fe5fbe511587657.pdfNo part of this book may be reproduced in any form, by photostat, microfilm, xerography or any other means,

KAPLAN LSAT PREP ______________________________________________________ LSAT Test X Explained: Section III

54 © K A P L A N

The Questions:

22. (B)Tricky, because the concept of elections never comes up directly. You need to infer thatthe answer will come out of ¶ 2, where the classical theory and its variants are mostcompletely defined, and so it does. All three variations, we’re told, “[conform] to acausal sequence characteristic of” the classical theory in which “some unusualcondition” is linked “to the generation of unrest” (lines 14-17). If an unusual conditionis therefore a necessary condition of the classical theory, then of course the electionproposed in the stem—an annual event—would not qualify.

(A) Contrary to (A), personalities can be studied under the classical theory, but in anyevent the content of the election is irrelevant to the issue of whether the classical theorywould apply.

(C), too, is an aspect of the election itself that has no bearing on how the classical theoryis described and applied. If voters’ psychological needs were relevant to the electionresults, presumably the classical theory could encompass them—if, that is, the theorycould be applied.

(D) picks up the phrase “momentous developments” from ¶ 1, where it’s used todescribe the black civil rights movement itself. No connection to the questionwhatsoever.

(E) need not be so—an election need not “entail two or more distinct socialmovements,” but so what if it did? Nowhere are we told that the presence ofsimultaneous social movements renders the classical theory inadequate or inapplicable.

• A major theme running through all of the passages on this PrepTest X has been:Know where in the passage each question is pointing you to. Even a question like22, which seems out of left field, must connect with the passage somehow. Thinkabout the passage logic, and move decisively.

23. (D)This question clearly sends you to ¶ 3, where the two models are described, if sketchily.The relative deprivation model is, we’re told, “only slightly different” from the risingexpectations model, and happily we needn’t spend much time figuring out why:correct choice (D) states the obvious.

(A) Neither model seems to have much predictive value. In any event, the two modelsare described insofar as each explains the civil rights movement of the past; we aren’ttold nearly enough to make a judgment on the predictions each would make.

(B) Au contraire, the relevance of a psychological explanation is something aboutwhich the two models in question agree. It’s what each is based on.

(C) No, each is described as explaining the same social change—the black civil rightsmovement. Each does so differently, which brings us back to (D).

Page 57: 10-PrepTest 10 Explsdl.keywin.org/1/e/1e1103afee7eee195fe5fbe511587657.pdfNo part of this book may be reproduced in any form, by photostat, microfilm, xerography or any other means,

KAPLAN LSAT PREP ______________________________________________________ LSAT Test X Explained: Section III

© K A P L A N 55

(E) is the one people choose if they work too hard, grabbing the phrase “system strain”from ¶ 2 (why, heaven only knows, because the two models aren’t described until a ¶later) and overthinking the “slight difference” briefly alluded to in line 29. In fact, so faras we can tell, the big difference between the two models is that while both modelsbelieve that protest began because of socioeconomic gains that predated the movement,“relative deprivation” brings in a second factor (lines 32-34). If anything, both modelsbelieve that socioeconomic status is relevant to the issues at hand.

• In many “detail questions,” if you work too hard to understand the detailschapter and verse, you waste a lot of time and end up choosing wrong answers.Don’t read the passage as if you were a real expert and truly interested in it! Readno more deeply than the questions and answers demand. Here, just the mostcursory review of ¶ 3 leads you to the right answer.

24. (E)“Their [i.e. the political theorists’] arguments rest on the conviction that, implicitly...”(lines 40-41) is a pretty good signal of an assumption that those theorists are making,and lines 40-43 sums it up. (E) spells out the underlying premise—namely, that thesocial psychologists must believe that social movements are better studiedpsychologically than politically. If that weren’t true—if social psychologists gave equalor greater weight to political motivations—then by the political theorists’ own logic, theclassical theory wouldn’t be as trivial or divorced from political analysis as it’s accusedof being.

(A) In no sense must the classical theory rely on conflicting motivations among theparticipants in a social movement. All could share the same motivation.The importantthing, as (E) says, is that under that theory, psychological motivations (of whatevertype) are paramount.

(B) errs in applying the criterion of social beneficence to the classical theory. Politicaltheorists do fault the psychological explanation of the civil rights movement, but not onthe grounds of whether society does or does not benefit.

(C) contradicts what we’re told about the classical theory, and there’s no reason tobelieve that political theorists share that error. To social psychologists, socioeconomicstrain was at the heart of this particular social movement (black civil rights), but theyneedn’t believe this to be true of every movement studied.

(D) sounds a lot like (E), but notice the subtle difference: (E) deals with the causes ofsocial movements. On this front, the political theorists imply that those ascribing toclassical theory rely on psychological rather than political motivations. (D), on the otherhand, deals with the ends of the movement participants. The fact that political theoristscriticize classical theorists for trivializing the political ends in their analysis of the causesof social movements doesn’t mean that they (political theorists) believe classicaltheorists assume that the political ends of the participants themselves are best analyzedthrough psychology.

Page 58: 10-PrepTest 10 Explsdl.keywin.org/1/e/1e1103afee7eee195fe5fbe511587657.pdfNo part of this book may be reproduced in any form, by photostat, microfilm, xerography or any other means,

KAPLAN LSAT PREP ______________________________________________________ LSAT Test X Explained: Section III

56 © K A P L A N

• The answers to inference and assumption questions in Reading Comprehensionare subject to The Denial Test, just as they are in Logical Reasoning.

25. (B)The first of two questions on the “better test” asks for an inference. Remember, the issueis whether the socioeconomic explanation of the civil rights movement holds up, andthe author has just gotten through arguing (lines 57-65) that any of the three “classical”models can be supported by different types of economic evidence. The author’s “bettertest” brings in a new issue, the press. How often do those evidence types correlate withthe civil rights protests and other activities described in the paper? Not much—whichweakens the case for any one of the three theories.

(A) Au contraire, the test leaves the theories no more proven than before.

(C) The test isn’t quite that devastating. All it tells us that these three social psychologicaltheories remain unconfirmed.

(D) No evidence. The press could’ve been totally objective.

(E) In no way does the test indicate this. Way beyond the question’s scope.

• Context is really important. If you recognize that the purpose of ¶ 5 is todemonstrate that, to the author, the classical theories are dubious, then (B) is theonly good choice.

26. (D)The author’s implicit point in the description of the “better test” (see lines 65-70 and theexplanation to Q. 25) is that if a classical theoretical model were a valid explanation ofthe civil rights movement, we would expect to see the evidence supporting that modelcorrelate well with movement events. But that assumes that the press adequatelyreported on all of the relevant movement activities. If the press didn’t do so, it’spossible that the real support for one of the theories never made it into the papers.

(B) Tempting if you read it too quickly. However, if all of the economic indicatorsrelevant to the movement and the classical theory were in fact reported by the press (andthey might have been), then the author’s “better test” would be unharmed. So thepossible holes in press coverage do not, in and of themselves, act to weaken the author’stest.

(C) Yes, they may, but such a contradiction would only be meaningful if the evidencelent support to more than one of the theories, in which case we might have troublesorting them out. But none of the three theories gains support from the test.

(D) distorts the thrust of the test, which is to compare the economic indicators with “thepace of reports about movement activity” (lines 64-65). No one “event,” movement-initiated or otherwise, is part of the test.

Page 59: 10-PrepTest 10 Explsdl.keywin.org/1/e/1e1103afee7eee195fe5fbe511587657.pdfNo part of this book may be reproduced in any form, by photostat, microfilm, xerography or any other means,

KAPLAN LSAT PREP ______________________________________________________ LSAT Test X Explained: Section III

© K A P L A N 57

(E) The issue isn’t whether the press, or anyone else, can keep pace with change, butrather how the press kept up with the black civil rights movement years ago and howthe reports of activity correlated with economic indicators.

• It’s worth noticing whether more than one question focuses on the same part ofthe passage, as 25 and 26 do here. You may be able to attack two for the price ofone.

27. (E)(E) recognizes both the three ¶’s that describe the thinking behind the socialpsychological theories, and the two ¶’s that point out weaknesses in those theories.

(A) is way too broad—you can’t get much further beyond the scope of this passage than(A). Also, there’s no sense that historians are this author’s target audience.

(B) The model is “classical,” not new, and it’s a model of social theory.

(C) Tempting, but the descriptions first of the social psychological theories, and then ofthe objections of political theorists, doesn’t qualify as a “shift,” but rather as a two-pronged discussion such as correct choice (E) describes.

(D) The only thing that evidently “unifies” the three classical theoretical models is thatnone of them is adequate to the task of describing why the black civil rights movementdeveloped. And like (B), (D) ignores the fact that they are theories of social movement,not “models”of same.

• Don’t be surprised to discover a very easy question residing as #27. Thetestmakers traditionally award those who persevere to the end of a section withsome very do-able questions. Whether it’s Reading Comp. or any of the othersections, make it your business to see that you get to the end of every section andgive every question a fair shake. It’s in your best interest to do so!

Page 60: 10-PrepTest 10 Explsdl.keywin.org/1/e/1e1103afee7eee195fe5fbe511587657.pdfNo part of this book may be reproduced in any form, by photostat, microfilm, xerography or any other means,

58 © K A P L A N

SECTION IV:LOGICAL REASONING

Page 61: 10-PrepTest 10 Explsdl.keywin.org/1/e/1e1103afee7eee195fe5fbe511587657.pdfNo part of this book may be reproduced in any form, by photostat, microfilm, xerography or any other means,

KAPLAN LSAT PREP ______________________________________________________ LSAT Test X Explained: Section IV

© K A P L A N 59

1. (A)The argument is pretty simple, summed up in the last sentence: Patients shouldn’t getcheckups unless they feel sick. (A) weakens the argument by showing that a patient whomay not feel ill might nevertheless benefit from an examination.

(B) doesn’t address the issue of whether patients who feel fine should go for a checkup.Furthermore, this choice deals with the amount of time a doctor spends on a checkup,while the argument is concerned with thoroughness.

(C) The argument discusses the doctor’s judgment of how thorough to be with a patient.Knowing that a patient is no more able than a physician to make that judgment doesn’tweaken that argument. Like (B), this choice doesn’t offer any reason why a seeminglyhealthy individual should go for a checkup.

(D) If anything, this strengthens the argument by presenting another good reason toavoid checkups.

(E) While some physicians might exercise the right amount of thoroughness, othersmight not. As long as there are some that don’t, the argument against “unnecessary”checkups still holds.

• Weakening an argument involves a direct attack on the argument’s conclusion ora critical piece of support/evidence.

• When weakening an argument, determine what is holding the argument together.Ask yourself, “which piece(s) of support are necessary to reach the statedconclusion.” Look for choices that contradict that piece of support, offer anotherinterpretation of that evidence, etc.

2. (B)One of the reasons given for healthy patients avoiding checkups is that some doctorsare over-thorough and hence likely to perform unnecessary tests. (B) points out apossible danger in these unnecessary tests and, in doing so, lends support to the notionthat patients who feel fine should avoid checkups.

(A) Fairly irrelevant; if anything, this weakens the argument by softening one of thenegative aspects of the over-thorough doctor.

(C) Those who don’t believe a physician who pronounces them healthy undoubtedlybelieve themselves to be ill. The argument, however, is aimed at those who feel fine.

(D) weakens the argument, in that it demonstrates a possible benefit of a completemedical checkup. In this case, the exhaustive testing might save a patient’s life, whereasthe passage views such testing as a hindrance.

(E) Another choice that leans in the other direction, by offering a reason why going tothe doctor isn’t so bad. Presumably this type of doctor would be thorough withoutburdening the patient with unnecessary uncomfortable and expensive tests. While it

Page 62: 10-PrepTest 10 Explsdl.keywin.org/1/e/1e1103afee7eee195fe5fbe511587657.pdfNo part of this book may be reproduced in any form, by photostat, microfilm, xerography or any other means,

KAPLAN LSAT PREP ______________________________________________________ LSAT Test X Explained: Section IV

60 © K A P L A N

may not be a very convincing weakener, it certainly doesn’t support the argumenteither.

• In strengthen the argument questions, look for the choice that makes theconclusion seem more likely or more believable.

• When a passage is followed by one weaken question and one strengthenquestion, you must remain open-minded; you have to see the argument from bothsides, much as you’ll be required to do in law school.

3. (C)The issue here is the different ways in which people pronounce words when reading asopposed to speaking off-the-cuff, and how that difference impacts on computer speechrecognition. A conclusion based on these observations should therefore address thisissue, and (C) flows logically; if the computers are programmed from read speech, theywill likely have a more difficult time decoding a different type of speech (i.e.,spontaneous).

(A) is a little too extreme; the computers probably could decode spontaneous speech,just not as well as words read off a page. Also, the passage deals with the problemsinherent in the programming today — it makes no mention of what is possible in thefuture.

(B) No mention of different accents was made — the statements deal with differingspeech patterns of the same person.

(D) Once again, too extreme — just because something is more likely to be a problemdoesn’t mean that it will never work.

(E) We’re told nothing to suggest that speech-recognizing computers are flawless.Besides, the scope of the argument covers computers in the immediate or near future(check it out).

• Be wary of choices that contain extreme-sounding language — “impossible,”“never” — unless similar terms are used in the argument.

• Note the difference between finding the author’s conclusion (stated in thepassage) and a conclusion derived from the statements in a stimulus (unstated;represents the next logical leap).

4. (D)The conclusion that there was no discrimination against qualified women is supportedby the single piece of evidence that half of those newly admitted were women. Theargument would crumble if (D) were not true. If women represented 75% of thequalified applicant pool, for example, but only 50% of those admitted, such statisticswould indicate possible discrimination.

Page 63: 10-PrepTest 10 Explsdl.keywin.org/1/e/1e1103afee7eee195fe5fbe511587657.pdfNo part of this book may be reproduced in any form, by photostat, microfilm, xerography or any other means,

KAPLAN LSAT PREP ______________________________________________________ LSAT Test X Explained: Section IV

© K A P L A N 61

(A) doesn’t address the issue of gender at all, and besides, the raw number of qualifiedapplicants is inconsequential, as the argument deals in percentages.

(B), (C) The proportion of skaters in the town that are male or female has no bearing onthe discrimination issue — the makeup of the applicant pool is what we’re mostinterested in. The key to rejecting both is seeing that both sidestep the issue ofqualifications — it’s qualified female applicants who may have been discriminatedagainst.

(E) This choice can be misleading; if at least half of the selection committee werefemale, one might be tempted to conclude that this information in fact strengthens thenotion that the club did not engage in discrimination. However, this is a valuejudgment that has no support in the argument, so the validity of the argument certainlydoesn’t depend on it.

• “Assumption” questions always involve a missing link in the logical chain ofevents. Sometimes you can paraphrase it yourself. Other times, check each choiceto see how badly the argument would be damaged if the idea in that choice werenegated.

• Don’t allow your own preconceptions to bias your decisions; base yourevaluations strictly on the logical components of the stimulus.

5. (B)The argument can be restated generally in the following form: When a group (girls) hasa particular attribute (education in single-sex secondary schools), its members tend tobe above average in some second attribute (academic achievement). Since a particularperson (Alice) has the extreme of the second attribute (the highest college grades), s/heprobably, therefore, has the first attribute. (B) parallels this structure exactly.

(A) The structure of the conclusion is “since the person has the first attribute, s/he musthave the second.” This is opposite from the original.

(C) and (E) simply make general statements and then follow up with generalizationswhich are not necessarily logical consequences. These can’t be parallel, however, sincethere’s no application to a specific person.

(D) The conclusion here is of the form “since the person lacks the first attribute, she willlikely lack the second.”

• All elements of the original argument must be present in its parallel. For example,if the original argument made a generalization to a specific case, a secondargument, no matter how similar in structure, cannot be parallel unless it makes acomparable generalization.

• Stay away from answer choices written about the same subject matter as theoriginal. This is an old trick of the testmakers, intended to catch those who arenot aware that they are to mimic the structure, not the content, of the stimulus.

Page 64: 10-PrepTest 10 Explsdl.keywin.org/1/e/1e1103afee7eee195fe5fbe511587657.pdfNo part of this book may be reproduced in any form, by photostat, microfilm, xerography or any other means,

KAPLAN LSAT PREP ______________________________________________________ LSAT Test X Explained: Section IV

62 © K A P L A N

6. (D)The argument here starts with the early stages of a trend (energy sources decreasing incarbon and increasing in hydrogen) and makes the quick jump to a future prediction(pure hydrogen as the energy source). The underlying principle will fill in the gapbetween the early trend and the end result. (D) describes a phenomenon that, if appliedto these facts, would explain why the energy source of the future will use no carbon andonly hydrogen.

(A) The argument suggests that the trend will continue in a single direction, movingsteadily towards a new state, i.e. using only hydrogen, whereas this choice indicates acircular pattern.

(B) This principle would suggest an evolution towards a fuel that contained equalparts carbon and hydrogen, which contradicts the scenario in the stimulus.

(C) The time it takes the stages of the process to unravel is irrelevant.

(E) is off the point as it attempts to resolve what is “best,” something that’s not stronglyimplied in the example. Furthermore, (E) describes a situation where a substance isgiven all of the best attributes in one shot, whereas the stimulus describes a slowdecrease in the carbon to hydrogen ratio over the course of many energy sourcetransitions.

• Principles and assumptions serve similar functions in an argument, but in asubtly different way: Assumptions are specific additional pieces of informationthat allow the conclusion to follow from the evidence, whereas principles aremore general statements that supply a logical justification for connecting theevidence with the conclusion. Nevertheless, the two question types can behandled in a similar way.

• The correct underlying principle will generally parallel the fact pattern closely.Read each choice against the facts until you find a match — and be bold inrejecting choices that veer significantly from the facts.

7. (A)Both X and Y appear to agree that preserving particular plant species is important, butthey feel this way for completely different reasons. X wants to preserve the plants sothey can be used as a natural resource from which important chemicals can be derived.Y, on the other hand, argues that they should be preserved so as to respect their right tosurvive. Choice (A) addresses this disparity in their reasons for advocatingconservation.

(B) Neither X nor Y discusses costs.

(C) Y doesn’t agree or disagree with the idea that it’s important to conserve naturalresources. Y’s argument is simply that the plant species are not, in fact, naturalresources.

Page 65: 10-PrepTest 10 Explsdl.keywin.org/1/e/1e1103afee7eee195fe5fbe511587657.pdfNo part of this book may be reproduced in any form, by photostat, microfilm, xerography or any other means,

KAPLAN LSAT PREP ______________________________________________________ LSAT Test X Explained: Section IV

© K A P L A N 63

(D) No point of contention here — both X and Y are in agreement that the extinction ofliving beings should be prevented.

(E) There is no indication as to how X feels about this, and we can infer that Y feels thatall non-human species, as living things, have no value as natural resources.

• When looking for the issue about which two parties disagree, make sure that youselect an issue that’s relevant to both arguments, not just one.

• In a two-person dialogue, reading the second person’s statement first usuallysaves time and reduces confusion. Here, after reading Y, you know that X isconcerned about “the good [species] can do us” before you even read X’speroration.

8. (C)X argues “to preserve for all time our natural resources” so that chemicals can beextracted from particular plants. If, however, using the plants for chemicals wouldactually eventually cause the plants’ extinction, then the whole effort to conserve themwill have been counterproductive. Since this negation of (C) causes the argument to fallapart, we can be confident that (C) is a assumption that’s crucial to the argument.

(A), (E) We can reasonably infer that X might believe these to be true; however, that’snot the same as saying that X’s argument relies on these pieces of information — his orher argument would be unaffected by the absence of either.

(B) is too broad, as “all living things” includes humans and animals and all sorts ofother groups that are outside the scope of the argument.

(D) The basis of X’s argument is to show that it’s in our best interest to preserve naturalresources, but whether this is the only way to advocate this position is irrelevant.

• When a question is directed to only one person of a two-person dialogue, makesure to focus only on that person’s argument, and not to be distracted by thepoints made by the other.

9. (D)Conclusion: Carpet manufacturers can only gain additional market share by acquiringcompetitors. (D) weakens this by showing an alternative way to increase market share:If some of the producers disappeared from the market, their customers could besnatched up by the surviving companies, thus increasing the survivors’ market share.

(A) The scope of this argument is limited to market share in the carpet industry. Anymention of any other products is outside the scope.

(B), (E) These choices tends to strengthen the notion raised in the first sentence, namelythat there’s relatively little room for growth in the market.

Page 66: 10-PrepTest 10 Explsdl.keywin.org/1/e/1e1103afee7eee195fe5fbe511587657.pdfNo part of this book may be reproduced in any form, by photostat, microfilm, xerography or any other means,

KAPLAN LSAT PREP ______________________________________________________ LSAT Test X Explained: Section IV

64 © K A P L A N

(C) The argument states that purchasing competitors is the only way to gain marketshare; however, this doesn’t guarantee that doing so will lead to success. It therefore hasno bearing on the argument.

• One way to cast doubt on a conclusion is to look for a choice that presents asituation that would turn the conclusion around: You think the carpet market’stapped? If (D) were true, there’d be lots of room for growth via aggressivemarketing.

• When attempting to cast doubt on a conclusion, or weaken an argument, theanswer will invariably either attack the conclusion directly or else will breakdown a piece of prime evidence.

10. (A)We’re told that one type of decision maker will sometimes advocate a position just tosee how others react to it rather than because of a commitment to that position. Thissupports (A), which states that the substance of what some decision makers say mightnot necessarily be consistent with the decisions they ultimately make.

(B) That some decision makers present contrary ideas (in order to test out what theyactually believe in) implies that they, at least, do have an idea of how they feel beforethey approach their advisers.

(C) The passage is based on decision making through discussion, so the nature of thedecision in this one is outside of the scope.

(D) Some decision makers may argue in favor of an idea they don’t believe in for thepurpose of confirming their suspicions, but this in no way suggests that they will endup settling for an idea in which they don’t believe.

(E) The decision makers seek to test their beliefs by listening to others. If the advisersknew the actual beliefs of the decision makers in advance, this would probably skewtheir advice towards what they think the speaker wants to hear, and thus defeat thedecision maker’s purpose.

• Don’t be tempted by choices that appear to be true statements unto themselves; ifthey don’t answer the specific question that’s asked, cross them off.

• Translate wordy choices into simpler manageable language, in order to evaluatetheir validity.

11. (A)The argument here is that there must be some additional explanation for Zeria’ssevering of diplomatic relations with Nandalo other than Nandalo’s poor record onhuman rights. (A) provides solid support for this argument by offering a plausiblealternative explanation for Zeria’s actions.

Page 67: 10-PrepTest 10 Explsdl.keywin.org/1/e/1e1103afee7eee195fe5fbe511587657.pdfNo part of this book may be reproduced in any form, by photostat, microfilm, xerography or any other means,

KAPLAN LSAT PREP ______________________________________________________ LSAT Test X Explained: Section IV

© K A P L A N 65

(B) focuses on added evidence that would suggest that Nandalo does violate humanrights, but it doesn’t deal with the argument that there’s more than human rights at thebasis of Zeria’s actions.

(C) Irrelevant; no comparison between Nandalo and Zeria with respect to this issue israised in the argument.

(D) We can’t infer that this is the additional reason for Zeria’s actions (they’re gonnaaccuse us, so let’s accuse them and sever relations), because there’s no way of knowingthat Zeria knew of Nandalo’s plan.

(E) The opposition party is outside the scope: The argument centers around the primeminister’s announcement and the author’s response to it.

• Be careful of choices that lend support to the wrong part of the argument.Showing that Nandalo in fact is a human rights offender is all well and good, butdoesn’t lend support to the entire argument. The choice that does support theentire argument will bolster the conclusion itself, or else forge a stronger linkbetween support and conclusion.

12. (D)We can generalize the structure of the argument as: Someone took action A (severingdiplomatic relations with Nandalo) in opposition to B (Nandalo’s human rightsviolations). However, action A has not been taken in other cases when B is present.Therefore, opposition to B can’t be the full reason that person took action A. (D) followsthis same structure: Armand took action A (declined invitation to dinner) inopposition to B (socializing with coworkers). However, he didn’t avoid socializing withcoworkers when he went to a movie with Maria. The conclusion is that there must besome additional explanation, other than hangups about fraternizing with coworkers,that explains his blowing off Helen’s invitation.

(A) Henry’s parents advocate action by one person for that person’s benefit while notpursuing the same action themselves. Although a “double standard,” their reasoningcan nevertheless be reconciled with their differing actions — their insistence could bemotivated solely by a concern for his health, even if they don’t take their own advice. (Inrhetoric we call this a tu quoque — “you do it too” — response. You didn’t have to knowthe Latin name, but if you recognized that that’s not what’s going on in the original, youcould have thrown (A) out on that basis.)

(B) Professor Walsh’s statement is obviously referring to the handwriting of herstudents; it’s not inconsistent that she would nevertheless rely on her own handwritingfor lectures, and no additional explanation is needed.

(C) James’ reasoning is inconsistent with other facts. However, it’s not parallel since itdoesn’t show any hypocrisy in James’ actions in regard to different circumstances.

(E) focuses on the reasoning behind a general proposition rather than an individual’sactions, so it can’t be parallel.

Page 68: 10-PrepTest 10 Explsdl.keywin.org/1/e/1e1103afee7eee195fe5fbe511587657.pdfNo part of this book may be reproduced in any form, by photostat, microfilm, xerography or any other means,

KAPLAN LSAT PREP ______________________________________________________ LSAT Test X Explained: Section IV

66 © K A P L A N

• Due to its sheer length, this is a good question to skip and return to later (if timepermits; if not, guess).

13. (B)In reading the argument, we find three parts: a general statement; an example easilyidentified by the structural signal “a case in point;” and a conclusion that flows from theinitial statement. From the structural signal in the second sentence, we can easilyidentify (B) as the correct choice.

(A) If the statement about Frenson were false, the argument could still hold true. Wewould then simply regard Frenson as being among the “few” who will go against theirown self-interest.

(C) There’s no specific measure being advocated; the author’s conclusion is about how aparticular type of measure would have to be enacted.

(D) The Frenson example does imply that incumbents have at least one advantage overchallengers, but that’s not the reason for the example. As we saw, its purpose in theargument is to illustrate the point in the first sentence, showing how a particularpolitician refused to vote against his own self-interest.

(E) Frenson is mentioned as evidence in favor of the argument for direct popular votes,not as a limit to it.

• Use structural signals to help you sort out the key parts of the argument. Thephrase “a case in point” in the second sentence lets you know that AugustFrenson is not being introduced as a new thought, but as an example of thestatement made in the previous sentence.

• Read critically! Even if the question doesn’t explicitly ask for it, always try toidentify the role each statement plays in the overall argument. Is it a premise?Support? Conclusion? Recognizing which statements serves which purpose willhelp you on every question.

14. (B)In this experiment, the researcher made the error of assuming that kinesthesia wassufficient for maze-learning simply because there was no evidence to the contrary. Sincethe researcher never tested this point, it’s possible that kinesthesia, by itself, is notsufficient while still being necessary. (B) illustrates this point by suggesting thepossibility that kinesthesia needs to be combined with at least one other sense.

(A) Any “small differences” observed by the researcher are irrelevant to the reasoningemployed. All that’s important is that the researcher reached a general conclusion fromfairly similar results.

(C), (D) First of all, neither of these choices can be determined from the data. Secondly,if they could, they would lean more in support of the argument that kinesthesia is

Page 69: 10-PrepTest 10 Explsdl.keywin.org/1/e/1e1103afee7eee195fe5fbe511587657.pdfNo part of this book may be reproduced in any form, by photostat, microfilm, xerography or any other means,

KAPLAN LSAT PREP ______________________________________________________ LSAT Test X Explained: Section IV

© K A P L A N 67

sufficient, rather than damage it. Thirdly, both contain aspects that are beyond thescope — the mechanisms of transference and response to nonkinesthetic stimulation.

(E) The biggest difference between this and (B) is that (B) correctly offers the interactionwith another sense as a possibility. Like the previous two choices, what’s stated here can’tactually be determined by the data, so it isn’t a strike against the researcher’s reasoning.

• Make sure you’re clear on the difference between necessary and sufficientconditions. A sufficient condition is one that in and of itself is sufficient(enough) for a result, while a necessary condition is required for a result but maybe only one of several other conditions. Consider this: “If I’m watching MelrosePlace, I must be watching the Fox Network.” That’s a true statement, becausewatching Melrose Place is sufficient to determine that one is watching Fox, whilewatching Fox is necessary in order to watch Billy and Amanda go through theirpaces.

• Notice how the wording of this question differs from the type that asks “whichone of the following, if true, would weaken the argument”? In the former, theflaw is self-contained in the stimulus. In the latter, you’re given new information,and then asked to apply it to the situation and evaluate whether it damages theargument. A subtle difference, but one worth noting nonetheless.

15. (B)The basis for the opposition to the law is found in the last sentence — the legislationwouldn’t just prevent criminals from purchasing handguns, but would prevent law-abiding citizens from doing likewise. The principle in (B) serves to justify thisopposition by stating that nothing should be done that would result in this situation.

(A) While the argument is founded on the notion that the rights of law-abiding citizensshould be protected, no comparison is made between the worth of their rights and therights of criminals.

(C) No; as a matter of fact the argument seems to suggest that a benefit would accrue tosociety as a result of the legislation (some criminals wouldn’t have access to guns), butthat benefit isn’t worth the restrictions it would inevitably place on non-criminals.

(D) doesn’t address the claim made by the opposition that the law will place restrictionson law-abiding citizens.

(E) The issue of adequate training is beyond the scope of the argument.

• A principle doesn’t support the argument unless it’s fully relevant. A choice thatonly partially touches on issues raised in the argument is insufficient to do thejob.

• Don’t be tempted by principles that seem consistent with the author’s viewpoint;they’re still wrong if they don’t lend direct support to the argument’s conclusion.

Page 70: 10-PrepTest 10 Explsdl.keywin.org/1/e/1e1103afee7eee195fe5fbe511587657.pdfNo part of this book may be reproduced in any form, by photostat, microfilm, xerography or any other means,

KAPLAN LSAT PREP ______________________________________________________ LSAT Test X Explained: Section IV

68 © K A P L A N

16. (D)Pretty simple one. The statement “The channels do show physical evidence of havingbeen formed quickly” is very strong support for choice (D).

(A) Running water caused the deep channels in volcanic rock in this case, which in noway implies that only running water can produce this result.

(B) The only mention of the river is its geographic proximity to the channels. Nothingin the argument suggests which was formed first.

(C) The argument suggests that geologists might, in fact, be able to predict the amountof heat necessary to melt a glacier quickly, but can’t fathom what phenomenon couldgenerate that much melting power.

(E) This characterizes the thinking that led to the initial rejection of the flood theory,but the last sentence is support for the notion that geologists now accept the physicalevidence even in light of the mysterious circumstances surrounding it.

• If the passage says that somebody doesn’t know X, the choice that says that thesame person doesn’t know Y is a completely different statement. Don’t let carelessreading force you to lose sight of this distinction.

17. (E)The paradox here is the scientific community’s acceptance of the flood theory without aclue as to what might have caused the quick melting of the glacier ice. (E) resolves theparadox by offering an alternative: If the flooding originated from the release of thetrapped water of the huge lake after the glacier’s retreat, then it’s no longer necessary tounderstand the ice-melting process in order to fully accept the flood theory.

(A) It’s already a given that water most likely caused the channels, so this choice doesnothing to resolve the paradox.

(B) So what? What do we know about those formations that will help us to reconcilethe flood theory? Nothing.

(C) There’s nothing in the argument that would change if multiple glaciers werepresent instead of a single one.

(D) All this does is explain why it’s more likely that channels form in volcanic rockthan in other types. It doesn’t, however, explain why the scientific community is nowwilling to accept the flood theory without solving the melting mystery.

• Before attempting to resolve a paradox, make sure you have a good grasp of whatthe paradox is.

• A paradox exists when there are two (or more) seemingly inconsistent statements.Resolving the paradox means reconciling the statements while allowing themboth still to be true.

Page 71: 10-PrepTest 10 Explsdl.keywin.org/1/e/1e1103afee7eee195fe5fbe511587657.pdfNo part of this book may be reproduced in any form, by photostat, microfilm, xerography or any other means,

KAPLAN LSAT PREP ______________________________________________________ LSAT Test X Explained: Section IV

© K A P L A N 69

18. (E)The argument here is that regular use of BTI will destroy mosquito larvae, whilepreserving the rest of the animal population in and around the pond. If these animalscounted on the mosquito larvae as a food source, then eliminating this source of foodcould have the effect of reducing — or even eliminating — the fish, frogs and beneficialinsects. (E), by stating that no such dependence exists, is necessary to maintain theargument, and is therefore a valid assumption.

(A) The connection between mosquito larvae and mosquitoes isn’t featured at all in theargument, so there’s no reason why the argument would necessarily depend on thispiece of information.

(B) isn’t relevant to the argument. Whether larvae need a single body or multiple bodiesof water to evolve shouldn’t change the effectiveness of BTI, and that’s the argument’ssole point.

(C) The argument never suggests that mosquitoes are the only insects that pond ownersmight want to eliminate, so even if there are hundreds of other insect pests that pondowners wish to get rid of, the argument is still unaffected.

(D) is also beyond the scope. We have no knowledge as to whether “strict adherence” isnecessary: The ad simply recommends using BTI regularly. Either way, this has nobearing on the argument.

• An assumption must stay within the scope in order to have enough relevance tobe a necessary component of the argument.

• As in real life, advertisements often make bold generalizations and skip criticalsteps in the logical process. Whether attached to a “flaw,” “weaken,” or“assumption” question, your job will involve identifying the missing step(s).

19. (A)There are two problems inherent in updating a will: First, it’s often difficult todetermine which, among several wills, is the most current. Secondly, the executordoesn’t know if there’s a newer will that hasn’t been found. The proposed solution willcertainly take care of the first problem but doesn’t deal with the second: The executorwill still be unsure of whether there is another will newer than the last one that wasfound. (A) points out this flaw: The author’s proposal solves half the problem whiletrying to put it over as a full solution.

(B) The argument only deals with prevention, not containment of the negative effects ofthe problem. No distinction between the two is necessary.

(C) According to the first sentence, the person responsible for the problem is the personwriting the will. The proposed solution places the responsibility of “fixing” theproblem with that same person. There’s no shift of responsibility, so (C) is out.

Page 72: 10-PrepTest 10 Explsdl.keywin.org/1/e/1e1103afee7eee195fe5fbe511587657.pdfNo part of this book may be reproduced in any form, by photostat, microfilm, xerography or any other means,

KAPLAN LSAT PREP ______________________________________________________ LSAT Test X Explained: Section IV

70 © K A P L A N

(D) The benefits of the proposed solution clearly address part of the problem, and thatpoint was outlined satisfactorily. The only error here is that part of the problem wasstill left unaddressed. The author’s reasoning isn’t flawed in his failure to consider thenegative consequences of the proposal.

(E) contradicts the argument. At the time of the proposed action, i.e. the writing of anew will, the information needed (the date and whether or not there is a previous will)would be available.

• When a author makes a proposal, he or she need not assume the burden ofshooting down every single negative aspect of it in order for the argument itselfto remain valid.

20. (E)The author describes the interrelationship between some bee species and various plantspecies. In particular, the relationship described is sometimes one of complete reliancein that some plants can only be pollinated by a particular type of bee, and some beescan only feed off of specific plants. If, as stated in the last sentence, the elimination ofcertain bee species could result in the extinction of the plant which relies on that bee,then the reverse, stated in choice (E), seems equally plausible.

(A) The relationship between plants and bees described wasn’t discussed in anhistorical framework; nothing suggests that flowering plants and bees appeared onEarth at precisely the same time.

(B) This choice confuses necessity and sufficiency. While the “sole pollinator” speciesmay be necessary to the plant’s survival, it’s not necessarily sufficient to guarantee itssurvival. The bees could be alive and well but some other factor, like a drought, forexample, could still kill the flowers.

(C) This is outside of the scope of the passage. All we’re told is that some bees canpollinate only a single species of plant. Nothing in the stimulus suggests whether otherbees can pollinate any species of plant.

(D) The error in this choice is the use of the word “most.” Again, we are told that aparticular phenomenon exists with respect to some plants. Whether those plants are inthe minority or the majority is not disclosed.

• Read choices critically with meticulous attention to detail. Choice (D) uses theword “most,” instead of some, which alone is the difference between a point anda wrong answer.

Page 73: 10-PrepTest 10 Explsdl.keywin.org/1/e/1e1103afee7eee195fe5fbe511587657.pdfNo part of this book may be reproduced in any form, by photostat, microfilm, xerography or any other means,

KAPLAN LSAT PREP ______________________________________________________ LSAT Test X Explained: Section IV

© K A P L A N 71

21. (E)We’re first presented with an ideal, followed by a particular project that doesn’tconform with the ideal. Next is a statement of why the ideal is not met. The authorconcludes then that the project is completely flawed because the ideal is not met. Theflaw is that while the space project doesn’t meet the scientific planning “ideal” (theshortcoming), that doesn’t necessarily imply that the project is doomed. Nothingsuggests why non-conformity to the ideal is fatal.

(A) There is no ad hominem attack (the Latin name for attacking a person rather than theperson’s logic) here.

(B) This argument is not circular — that is, its evidence and conclusion aren’tfunctionally identical. We’re given a statement of principle, followed by a non-conforming example. The conclusion is a new statement, different from the originalpremise. It may be poor logic (in fact it is poor logic), but at least it’s different from theevidence.

(C) If any fault is assigned to the project planners, it’s only for creating a new goal mid-stream, not for failing to foresee the end of the Cold War.

(D) The argument here is not self-contradictory. It’s weak, insofar as insufficient causeis given for scrapping the project, but as an argument it’s fairly direct andunambiguous.

• Make sure you understand the testmakers’ favorite argumentative flaws, such asthe ad hominem attack and circular reasoning (represented by (A) and (B),respectively). Familiarity with these flaws allows you to eliminate these twochoices quickly.

22. (E)The first two sentences are actually strict formal statements: Experts are the only oneswho can understand Patrick’s behavior, and experts are unable to be certain of solvinghis problems. If Charles is certain of being able to solve Patrick’s problem, then he can’tpossibly be an expert (sentence 2). And if he’s not an expert, then he can’t possiblyunderstand Patrick’s behavior (sentence 1).

(A), (B) Nothing allows us to conclude that Patrick isn’t an expert in psychology. Hemay be, in which case he’d have a shot at understanding his own bizarre behavior.

(C) Patrick wants to devise a solution, but we can’t infer whether he feels certain ofdoing so, because the “certainty rule” applies only to people attempting to solve otherpeople’s problems.

(D) contradicts the passage. Since experts are the ones who can’t be certain of solvingthe behavior problem, being an expert would be a strike against Charles’s offering aviable solution.

Page 74: 10-PrepTest 10 Explsdl.keywin.org/1/e/1e1103afee7eee195fe5fbe511587657.pdfNo part of this book may be reproduced in any form, by photostat, microfilm, xerography or any other means,

KAPLAN LSAT PREP ______________________________________________________ LSAT Test X Explained: Section IV

72 © K A P L A N

• Occasionally, passages will contain statements that have no bearing on thepassage. Wrong choices will often focus on these extraneous statements; don’tpanic if the passage appears to contain statements that could just as easily havebeen left out.

• Some Logical Reasoning statements bear a remarkable resemblance to LogicGames rules. And much like Logic Games, the ability to form deductions willusually lead to the right answer.

23. (D)The logical progression here is from famine to decreased population to increasedwages, thanks to supply and demand. The Irish potato famine is offered as an exceptionto this generalization. (D), however, doesn’t help to explain the exception: Even if thebirth rate increased in the years that followed, that would have no immediate effect onthe work force — at least not in the following decade. We would therefore still expect adecreased work force and a rise in average wages.

(A), (B) If the labor force didn’t decrease, then it’s easy to explain the lack of wageincreases following the famine. If (A) were true, then it’s plausible that the post-faminework force stabilized more quickly than in other famine situations. (B) explains how thefamine may have had a massive effect on the overall population, but not nearly thesame effect on the work force.

(C) Back to supply and demand: If technology reduced the demand for labor, then thatcould explain how a smaller labor force would still be unable to command the higherwages than were otherwise typical in post-famine situations.

(E) Such governmental control resulting in artificially low wages could, in itself,explain this particular exception to the generalization.

• When seeking to explain how something might have occurred, your job is torecognize choices that are plausible. They don’t have to provide completeexplanations, but just move the argument in the right direction.

• When asked to find which choice least explains an event, you should check all ofthe choices. One should stick out like a sore thumb, next to the others that do amore complete job. Don’t split hairs in such situations.

24. (C)Two investments are referred to: the “most profitable” investment and “any particular”investment. In this model, an investment must be one or the other — it will either be themost profitable or else be somewhat less profitable (nothing can be more profitable thanthe most profitable). The circumstance described indicates that the investment inquestion is not the most profitable; if it were, then the decline would be equal to, andnot more than, the percentage defined. Therefore, the particular investment referred tomust be one that is less profitable than the most profitable investment.

Page 75: 10-PrepTest 10 Explsdl.keywin.org/1/e/1e1103afee7eee195fe5fbe511587657.pdfNo part of this book may be reproduced in any form, by photostat, microfilm, xerography or any other means,

KAPLAN LSAT PREP ______________________________________________________ LSAT Test X Explained: Section IV

© K A P L A N 73

(A) If the rate of inflation had risen, the percentage arrived at by comparing the mostprofitable rate of return with inflation would be larger than it is, but we’re given noindication that that’s where the passage is leading.

(B) The “value” of an investment is determined relative to the rate of inflation. Even ifan investment remained at a constant level of profitability, if inflation soared, the relativevalue of that investment would decrease. It’s purely a matter of semantics, buttechnically, under these circumstances, we can’t conclude that the investment is less“profitable,” only less “valuable.”

(D), (E) As with (A), nothing suggests a change in the rate of return on the mostprofitable investment, or in the most profitable investment itself; the argument isleading towards a conclusion about the “particular investment.”

• Always try to fill in the blank on your own before trying out the choices. It iseasier to find the right choice if you already know what you’re looking for.

• If you have trouble conceptualizing these types of numerical/percentage relatedstimuli, skip them for the time being and return to them later if time permits. Itmakes no sense to stare for two or three minutes at a problem that you still mayend up not grasping — a poor return on your investment of time.

25. (C)This argument begins by describing an eighteenth-century theory, and continues byasserting that a “well-respected physicist” has deemed the theory to be incoherent. Theauthor then rejects the theory on the grounds that it’s incoherent. The only support forrejecting the theory is the statement by the physicist. (C), therefore, best describes themethod of argumentation.

(A) Although there is some technical terminology in the argument (motion is absolute,object’s position over time), it’s fairly simple and certainly not used as a means ofargumentation.

(B), (D) No experimental results were used in the argument; the sole support for theargument is the statement by a single authority.

(E) is way off the mark. The conclusion has nothing to do with accepting a theory about“all regions of space” based on a generalization. In fact, the author rejects a theory asincoherent based solely on one “expert’s” opinion.

• Any stimulus with a philosophical bent is likely to be verbose. Just like inReading Comp, getting through these passages is often a matter of wading pastthe wordiness and complexity and getting to the crux of the argument.

• There aren't many formal labels for argumentative techniques that appear on theLSAT. Get familiar with the most common ones, and practice recognizing them inany stimulus you read.

Page 76: 10-PrepTest 10 Explsdl.keywin.org/1/e/1e1103afee7eee195fe5fbe511587657.pdfNo part of this book may be reproduced in any form, by photostat, microfilm, xerography or any other means,

I.N. LL3099 Rev.B Printed in the USA